Está en la página 1de 92

NUMBER SYSTEM

Prime Numbers: Numbers which are not divisible by any number


other then 1 and itself are called Prime numbers.
E.g. 2, 3, 5, 7,.
Note: Any prime number greater than 3 can be written as 6n+/_1,
where n is a natural number.

Steps to find whether a given number is prime number or
not:
1. Find the least positive integer, such that a
2
> given
number.
2. Test the divisibility of given number by every prime number
that is less than a.
3. The given number is a prime number only if it is not
divisible by any of these primes.

E.g. Check whether 923 is a prime number or not?
1. 923 lies between 900 and 961 which are perfect squares
having square roots 30 and 31 respectively.
2. Prime numbers less than 31 are 2,3,5,7,11,13,17,19,23,29.
3. 923 are not divisible by any of these numbers and hence it
is a prime number.

Co-prime Numbers: Numbers which do not have any common
factor other than 1 are called Co-prime numbers.
E.g. (4, 15), (9, 22), (12, 29),

Note:
1. 1 is neither prime nor composite.
2. 2 is an even prime number.
3. Co-prime numbers can be prime or composite numbers.
4. Any two prime numbers are always Co-prime numbers.
5. Any two consecutive positive integers are always co-primes.

Perfect Numbers: If the sum of the factors of a given number is twice the
number, the number is said to be a Perfect number.
E.g. Factors of 6 = 1, 2, 3, 6 and 1 + 2 + 3 + 6 = 12



Note:
1. 6, 28, 496, 8128,. are perfect numbers.
2. The only four digit perfect no is 8128
3. So far only 27 perfect nos are identified.

UNIT DIGITS: Any given number N, the unit digit will necessarily
be one of the digits from 0 to 9. When these are raised the unit digit
place follows a cyclic pattern.
The cyclical pattern is
0---0
1---1
2---2,4,8,6,2,4,8,6
3---3,

MULTIPLICATION TIPS:

1. For multiplication of a given number by 9, 99, 999, etc., that is by 10
n
1, the easy way is:
Put as many zeros to the right of the multiplicant as there are nines in the multiplier and from
the result subtract the multiplicant and get the answer.
E.g. Multiply 2893 by 99.

Sol:2893 x 99 = 2893 (100 1) = 289300 2893 = 286407.

2. For multiplication of a given number by 11, 101, 1001, etc., that is by 10
n
+ 1, the easy way
is: Place n zeros to the right of the multiplicant and then add the multiplicant to the number so
obtained.

E.g. Multiply 3782 by 11.

Sol: 3782 x 11 = 3782 (10 + 1) = 37820 + 3782 = 41602.

3. For multiplication of a given number by 15, 25, 35, etc. Double the multiplier and then
multiply the multiplicant by this new number and finally divide the product by 2.

E.g. Multiply 5054 x 15 = (5054 x 30) = (151620) = 75810

4. For multiplication of a given number by 5, 25, 125, 625, etc., that is, by a number which is
some power of 5. Place as many zeros to the right of the multiplicant as is the power of 5 in
the multiplier, then divide the number so obtained by 2 raised to the same power as is the
power of 5.

E.g. 2982 x 5 = 29820/2 = 14910
5739 x 25 = 573900/2
2
= 143475


a) No. of factors of a given number: If .....
r q p
c b a N = then the number of factors of N = (p + 1)
(q + 1) (r + 1)., where a, b, c are prime factors of N and p, q, r,. are positive integers.

E.g. Find the number of factors of 24.
Sol:24 =
1 3
3 2
The number of factors of 24 = (3 + 1) (1 + 1) = 8.



b) Sum of the factors of a given number: If .....
r q p
c b a N = then the sum of the factors of N =
......... ..........
1 c
1 c
1 b
1 b
1 a
1 a
1 r 1 q 1 p


+ + +
where a, b, c are prime factors of N and p, q, r,. are
positive integers.

E.g. Find the sum of the factors of 24.
Sol:24 =
1 3
3 2
Sum of the factors of 24 = . 60
1 3
1 3
1 2
1 2
1 1 1 3
=


+ +

c) No. of ways of expressing a given number as a product of two factors:
If .....
r q p
c b a N = where a, b, c are prime factors of N and p, q, r,. are positive integers then
the number of ways in which N can be expressed as product of two factors = { } )..... 1 r )( 1 q )( 1 p (
2
1
+ + + .
E.g. Find the number of ways of expressing 48 as a product of two factors.
Sol:48 =
1 4
3 2
No. of ways = { } { } 5 ) 1 1 )( 1 4 (
2
1
) 1 q )( 1 p (
2
1
= + + = + + .
d) No. of ways of expressing a given number which is a perfect square as a product of two factors:

If ..... c b a N
r q p
= where a, b, c are prime factors of N and p, q, r,. are positive integers then the
number of ways in which N can be expressed as product of two factors = { } 1 )........ 1 r )( 1 q )( 1 p (
2
1
+ + + + .
E.g. Find the no. of ways of expressing 36 as a product of two factors.
Sol:36 =
2 2
3 2
No. of ways = { } { } 5 1 ) 1 2 )( 1 2 (
2
1
1 ) 1 q )( 1 p (
2
1
= + + + = + + + .

Solved Examples

1. On dividing 64652 by a certain number, the quotient is 101 and the remainder is 12. Find
the divisor.

Sol: Here, the required number is divisor.

Divisor =
Quotient
Remainder - Dividend


= 640
101
64640
101
12 64652
= =



2. A number when divided by 160 leaves a remainder 52 and the quotient is 15. Find the
number.

Sol: Here, the required number is dividend.
Dividend = (Divisor x Quotient) + Remainder
= (160 x 15) + 52
= 2452

3. Find the least number of 5 digits which is exactly divisible by 642.



Sol: The least number of 5 digits is 10,000.
Dividing this number by 642, the remainder is 370.
So, the required number is 10,000 + (642 - 370) = 10272.

4. Find the greatest number of 5 digits which is exactly divisible by 642.

Sol: The greatest number of 5 digits is 99,999.
Dividing this number by 642, the remainder is 489.
So, the required number is 99,999 - 489 = 99510.

5. Find the number nearest to 14800 which is exactly divisible by 245.

Sol: The remainder on dividing 14800 by 245 is 100.
So, the number required number = 14800 100 = 14700 which is exactly divisible by
245.

6. Find whether 577 is a prime number.

Sol: (24)
2
= 576 < 577 and (25)
2
= 625 > 577
n = 25
Prime numbers less than 25 are 2, 3, 5, 7, 11, 13, 17, 19 and 23.
Since, 577 is not divisible by any of these numbers, it is a prime number.

7. How many numbers up to 531 are divisible by 15?

Sol: Divide 531 by 15.
531 = 35 x 15 + 6
The quotient is the required number and here it is 35.
So, there are 35 numbers up to 531 are divisible by 15.

8. How many numbers up to 200 are divisible by 5 and 7 together?

Sol: L.C.M. of 5 and 7 = 35.
Divide 200 by 35.
200 = 5 x 35 + 20
The quotient is the required number and here it is 5.
So, there are 5 numbers up to 200 are divisible by 35.

9. Find the number in the unit place in
59
) 729 ( .
Sol: 9 729 ) 1 (... 729 ) 729 ( ) 729 (
58 59
= = = in the unit place.

10. Find the number in the unit place in
42
) 98 ( .
Sol: 4 ) 4 (... ) 6 (... ) 98 ( ) 98 ( ) 98 (
2 10 4 42
= = =

in the unit place.

11. Find the number in the unit place in
36
) 636 ( .
Sol: 6 ) 6 (... ) 636 (
36 36
= = in the unit place.

12. Convert 0.4444.. into a rational number.
Sol: Let x = 0.4444..(1)
Since 1 digit (4) is repeating multiply equation 1 on both sides by
1
10 .
10 x = 4.4444.(2)
Subtract Equation 1 from 2 on both sides

10 x = 4.4444.
- x = 0.4444.


----------------------
9 x = 4.0000.
----------------------
.
9
4
x 4 x 9 = =

13. Convert 5.626262.. into a rational number.
Sol: Let x = 5.626262.. (1)
Since 2 digits (62) is repeating multiply equation 1 on both sides by
2
10 .
100 x = 562.6262.(2)
Subtract Equation 1 from 2 on both sides

100 x = 562.6262.
- x = 5.6262..
---------------------------
99 x = 557.0000.
---------------------------
99x = 557 x =
99
557


L.C.M. AND H.C.F.

Common Multiple: A common multiple of two or more numbers is a number which is exactly
divisible by each one of them.
E.g. 32 is a common factor of 8 and 16

Least Common Multiple (L.C.M): The least multiple among all the common multiples of given
numbers is called Least Common Multiple.

Methods of finding L.C.M.

1. Method of Prime Factors
a. Resolve each given number into prime factors.
b. Take out all the factors with highest powers that occur in given
numbers.
c. Find the product of these factors. This product will be L.C.M.

E.g. Find the L.C.M. of 12, 14 and 20.

12 = 2
2
x 3
14 = 2 x 7
20 = 2
2
x 5

So, the L.C.M. = 2
2
x 3 x 5 x 7 = 420

2. Method of Division

E.g. Find the L.C.M. of 12, 15, 18 and 20.


So, the L.C.M. = 2 x 2 x 3 x 5 x 3 = 180



Common Factor: A common factor of two or more numbers is a number which
divides each of them exactly.
E.g. 4 is a common factor of 8 and 12

Highest Common Factor (H.C.F): Highest common factor of two or more
numbers is the greatest number that divides each one of them exactly. It is also
called Greatest Common Divisor or Greatest Common Measure.

Methods of finding H.C.F.

1. Method of Prime Factors

E.g. Find the H.C.F. of 50 and 70

Sol: 50 = 2 x 5 x 5
70 = 2 x 5 x 7
Common factors are 2 and 5.
So, H.C.F. = 2 x 5 = 10


2. Method of Division

E.g. 1. Find the H.C.F. of 3332, 3724.

Sol:


So, the H.C.F. of 3332, 3724 is 196.

E.g. 2. Find the H.C.F. of 10, 15 and 23.

Step 1: First find the H.C.F. of 10 and 15. It is 5
Step 2: Then find the H.C.F. of this 5 and 23. It is 1.
So, the H.C.F. of 10, 15 and 23 is 1.
Note:

1. L.C.M. and H.C.F. of fractions





2. Product of two numbers = L.C.M. of two numbers x H.C.F.
of two numbers.



3. To find the greatest number that will exactly divide a, b
and c, simply find the H.C.F. of a, b and c.

4. To find the greatest number that will divide a, b and c
leaving remainders x, y and z respectively, find the H.C.F.
of (a x), (b y) and (c z).

5. To find the least number which is exactly divisible by a, b
and c, simply find the L.C.M. of a, b and c.

6. To find the least number when divided by a, b and c
leaving remainders x, y and z respectively, find the (L.C.M.
of a, b and c) k, where k = (a x) = (b y) = (c z).

7. To find the least number which when divided by a, b and c
leaves the same remainder r in each case, find (L.C.M. of
a, b and c) + r.

8. Two numbers when divided by a certain divisor give
remainders
1
r and
2
r . When their sum is divided by the
same divisor, the remainder is
3
r . Then the divisor is given
by
3 2 1
r r r + .

9. A number on being divided by
1
d and
2
d successively leaves
the remainders
1
r and
2
r , respectively. If the number is
divided by
1
d x
2
d , then the remainder = (
1
d x
2
r +
1
r ).

10. To find the greatest number that will divide x, y and z
leaving the same remainder r in each case:

Case 1: When the value of remainder r is given
Required number = H.C.F. of (x r), (y r) and (z
r).

Case 2: When the value of remainder is not given
Required number = H.C.F. of |(x y)|, |(y z)| and
|(z x)|.



11. To find the n-digit greatest number which when divided
by x, y and z:
a. Leaves no remainder i.e. exactly divisible
Step 1: Find the L.C.M. of x, y and z. Let it be L.
Step 2: Divide the n -digit greatest number by this L. Let the
remainder be R.
Step 3: Required number = (n-digit greatest number R).

b. Leaves remainder k in each case:
Required number = (n-digit greatest number R) + k.

12. To find the n-digit smallest number which when divided
by x, y and z:

a) Leaves no remainder i.e. exactly divisible
Step 1: Find the L.C.M. of x, y and z. Let it be L.
Step 2: Divide the n -digit smallest number by this L. Le the
remainder be R.
Step 3: Required number = n-digit smallest number + (L
R).

b) Leaves remainder k in each case:
Required number = n-digit smallest number + (L R) + k.

Solved Examples

1. Find the greatest number that will exactly divide 200 and 310.

Sol: The required number = H.C.F. of 200 and 310 = 10.

2. Find the greatest number that will divide 148, 246 and 623 leaving remainders 4, 6 and 11
respectively.

Sol: The required number = H.C.F. of (148 4), (246 6), and (623 11)
= H.C.F. of 144, 240, 612 = 12.

3. Find the smallest number that is exactly divisible by 45, 63 and 50.

Sol: Required number = L.C.M. of 45, 63 and 50 = 3150.

4. Find the least number which when divided by 36, 48 and 64 leaves the remainders 25, 37
and 53 respectively.



Sol: (36 25) = (48 37) = (64 53. = 11
Required number = (L.C.M. of 36, 48 and 64) 11
= 576 11 = 565

5. Find the least number which when divided by 12, 16 and 18, will leave the remainders 7 in
each case.

Sol: Required number = (L.C.M. of 12, 16 and 18) + 7
= 144 + 7 = 151

6. Find the greatest number which will divide 772 and 2778 so as to leave the remainder 5 in
each case.

Sol: Required number = H.C.F. of (772 5) and (2778 5)
= H.C.F. of 767 and 2773 = 59.

7. Find the greatest number which on dividing 152, 277 and 427 leaves equal remainder.

Sol: Required number =
H.C.F. of |(152 277)|, |(277 - 427)|, |(427 152)|
= H.C.F. of 125, 275 and 150 = 25.

8. Find the greatest number of 4 digits which, when divided by 12, 18, 21 and 28 leaves 4 as a
remainder in each case.

Sol: L.C.M. of 12, 18, 21 and 28 = 252.
Greatest 4-digit number = 9999.
The remainder when 9999 is divided by 252 = 171
So, the required number = (9999 171) + 4 = 9832.

9. Find the greatest number of 4 digits which, when divided by 12, 15, 20 and 35 leaves no
remainder.

Sol: L.C.M. of 12, 15, 20 and 35 = 420.
The remainder when 9999 is divided by 420 = 339
So, the required number = (9999 339) = 9660.

10. Find the least number of 4 digits which is divisible by 2, 4, 6 and 8.

Sol: L.C.M. of 2, 4, 6 and 8 is 24.
The least number of 4 digits = 1000
The remainder when 1000 divided by 24 = 16.
So, the required number = 1000 + (24 16) = 1008.

11. Find the smallest number of 4 digits when divided by 12, 18, 21 and 28 leaves remainder 5
in each case.

Sol: L.C.M. of 12, 18, 21 and 28 = 252
The least number of 4 digits = 1000
The remainder when 1000 divided by 252 = 244.
So, the required number = 1000 + (252 244) + 5 = 1013.

12. Two numbers when divided by a certain divisor give remainders 16 and 12 respectively.
When their sum is divided by the same divisor, the remainder is 4. Find the divisor.

Sol: Required divisor = 16 +12 4 = 24.

13. A number on being divided by 10 and 11 successively leaves the remainders 5 and 7,
respectively. Find the remainder when the same number is divided by 110.



Sol: Required remainder = 10 x 7 + 5 = 75.

14. Find the least number which when divided by 8, 10 and 15 leaves the remainders 3, 5 and
10, respectively.

Sol: Here, 8 3 = 10 5 = 15 10 = 5
L.C.M. of (8, 10, 15) = 120
The required least number = 120 5 = 115.

PERCENTAGE

Percent: The term per cent means per hundred or for every hundred. The word is derived from
the Latin word per centum.
Percentage: A fraction whose denominator is 100 is called a percentage.
Rate per cent: The numerator of the fraction is called rate per cent.
E.g.
100
5
and 5 percent means the same thing i.e. 5 parts out of every hundred parts.

Basic Formulae:
1. To convert any fraction
n
1
into a rate per cent, multiply it by100 and put % sign i.e. % 100
n
1

E.g. What percentage is equivalent to
4
3
?
Sol:
4
3
x 100 = 25%
2. To convert a per cent into a fraction, drop the per cent sign and divide the number by 100.

E.g. What fraction is 8
3
1
%?
Sol: 8
3
1
% =
3
25
=
3
25
x
100
1
=
12
1


3. x% of a given number (N) =
100
x
x N
E.g. 75 % of 800 = ?
Sol: 75 % of 800 =
100
75
x 800 = 600
4. If A is x % more than that of B, then B is less than that of A by
(


+
100
x 100
x
%.
5. If A is x % less than that of B, then B is more than that of A by
(

100
x 100
x
%.
6. If A is x % of C and B is y % of C, then A =
y
x
x 100% of B.
7. If two numbers are respectively x % and y % more than a third number, then the first
number is
(

+
+
100
y 100
x 100
% of the second and
the second number is
(


+
+
100
x 100
y 100
% of the first.
8. If two numbers are respectively x % and y % less than a third number, then the first number
is
(

100
y 100
x 100
% of the second and


the second number is
(

100
x 100
y 100
% of the first.
9. If the price of a commodity increases by N%, then the reduction in consumption so as not to
increase the expenditure is
(


+
100
N 100
N
%.
10. If the price of a commodity decreases by N%, then the increase in consumption so as not to
decrease the expenditure is
(

100
N 100
N
%.
11. If a number is changed (increased/decreased) successively by x % and y % then net %
change is given by
(

+ +
100
xy
y x % which represents increase or decrease in value according
as the sign is +ve or ve.

Note: If x and y indicates decrease in percentage, then put ve sign before x and y else +ve
sign.

12. If the population of a town (or the length of a tree) is P and its annual increase is r%, then:
(i) Population (or length of tree) after n years = .
100
r
1 P
n
|
|
.
|

\
|
+
(ii) Population (or length of tree) n years ago = .
100
r
1
P
n
|
|
.
|

\
|
+

13. If the population (or value of a machine in rupees) is P and annual decrease (or depreciation)
is r%, then
(i) Population (or value of machine) after n years = .
100
r
1 P
n
|
|
.
|

\
|

(ii) Population (or value of machine) n years ago = .
100
r
1
P
n
|
|
.
|

\
|


14. If a number K is increased successively by x % followed by y % and z %, then the final value
of K will be
K
(

+
100
x
1
(

+
100
y
1
(

+
100
z
1

15. In an examination, the minimum pass percentage is x%. If a student scores y marks and fails
by z marks, then the maximum marks in the examination is
x
) z y ( 100 +
.

16. In an examination a % and b % students respectively fail in two different subjects while c %
students fail in both the subjects, then the percentage of students who pass in both the
subjects will be (100 ( a + b - c ))%.


SOLVED EXAMPLES

1. If Srujana salary is 20% more than that of Deepa, then how much percent is Deepas salary
less than that of Srujana?

Sol: Here, x = 20.
Required answer =
(


+
100
x 100
x
%


=
(


+
100
20 100
20
%
=
(

6
100
% =
6
4
16 % =
3
2
16 %.
2. If Anitas income is 30% less than that of Saritha, then how much percent is Sarithas income
more than that of Anitha?

Sol: Here, x = 30.
Required answer =
(

100
x 100
x
%
= 100
30 100
30
%
=
(

7
300
% =
7
6
42 %.
3. If A is 25% of C and B is 30% of C, then what percentage of A is B?

Sol: A = 25% of C and B = 30% of C

A
B
=
25
30

B =
25
30
x 100% of A
= 120% of A

4. Two numbers are respectively 25% and 50% more than a third number. What percent is the
first of the second?

Sol: Here, x = 25 and y = 50
So, First number =
(

+
+
100
y 100
x 100
% of the second
=
(


+
+
100
50 100
25 100
% of the second
=
6
500
% of the second
=
6
2
83 % =
3
1
83 % of the second.
5. Two numbers are respectively 20% and 32% less than a third number. What percent is the
second of the first?

Sol: Here, x = 20 and y = 32
So, Second number =
(

100
x 100
y 100
% of the first
=
(

100
20 100
32 100
% of the first
= 85% of the first.

6. If the price of a commodity increases by 50%, find how much percent its consumption be
reduced so as not increase the expenditure.
Sol: Reduction in consumption =
(


+
100
N 100
N
%
=
(


+
100
50 100
50
%


=
(

3
100
% =
3
1
33 %.
7. If the price of a commodity decreases by 50%, find how much percent its consumption be
increased so as not decrease the expenditure.
Sol: Increase in consumption =
(

100
N 100
N

=
(

100
50 100
50
%
= 100%.

8. If the salary of Mr. Shashi is first increased by 18% and thereafter decreased by 15%, what is
the net change in his salary?

Sol: Here, x = 18 and y = -15
So, the net % change in the salary =
(

+ +
100
xy
y x %
=
(


100
) 15 )( 18 (
15 18 %
=
(


100
) 15 )( 18 (
15 18 %
= 0.3%.
Since the sign is +ve, there is an increase in the salary of person by 0.3%.

9. The population of a town is decreased by 20% and 40% in two successive years. What
percent population is decreased after two years?

Sol: Here, x = - 20 and y = - 40
So, the net % change in population =
(

+ +
100
xy
y x %
=
(


+
100
) 40 )( 20 (
40 20 %

=
(

+
100
800
60 %

= - 52%.

Since the sign is -ve, there is decrease in population after two years by 52%.

10. If the side of a square is increased by 10%, its area increased by k%. Find the value of k.

Sol: Area of square = side x side
So, net % change in area =
(

+ +
100
xy
y x %
=
(

+ +
100
) 10 )( 10 (
10 10 % [Take x, y = 10]
= 21%
Hence, the area is increased by 21%.
Here, k = 21.

11. The radius of a circle is increased by 4%. Find the percentage increase in its area.



Sol: Area of circle = x radius x radius
So, net % change in area =
(

+ +
100
xy
y x %
=
(

+ +
100
) 4 )( 4 (
4 4 % [Take x, y = 4]
=
(

+
100
16
8 % =
25
4
8 %.
12. The tax on a commodity is diminished by 12% and its consumption increases by 10%. Find
the effect on revenue.

Sol: Revenue = tax x consumption
So, net % change in revenue =
(

+ +
100
xy
y x %
=
(


+ +
100
) 10 )( 12 (
10 12 % [Take x = - 12, y = 10]
=
(


100
120
2 % = - 3.2%.
So, the revenue decreases by 3.2%.

13. The population of a town increases by 4% annually. If its present population is 12500, what
will it be in 2 years time?

Sol: Here, P = 12500, r = 4 and n = 2.
Population after n years = .
100
r
1 P
n
|
|
.
|

\
|
+
Population after 2 years =
2
100
4
1 12500
|
|
.
|

\
|
+
=
2
25
1
1 12500
|
|
.
|

\
|
+
=
2
25
26
12500
|
|
.
|

\
|


=
25
26
25
26
12500
= 13520.

14. The population of a town increases by 10% annually. If its present population is 12100, what
it was 2 years ago?

Sol: Here, P = 12100, r = 10 and n = 2.
Population n years ago =
n
100
r
1
P
|
|
.
|

\
|
+

Population 2 years ago =
2
100
10
1
12100
|
|
.
|

\
|
+

=
2
10
11
12100
|
|
.
|

\
|
=
10
11
10
11
12100




= 10 10
11 11
12100

= 10000.

15. The population of a town is 100000. It increases by 5% during the first year. During the
second year, it decreases by 10% and increases by 15% during the third year. What is the
population after 3 years?

Sol: Here, P = 100000, x = 5, y = - 10 and z = 15
So, Population after 3 years = P
(

+
100
x
1
(

+
100
y
1
(

+
100
z
1
= 100000
(

+
100
5
1
(


100
10
1
(

+
100
15
1
= 108675
100 100 100
115 90 105 100000
=






16. In an examination, a student must get 50% marks to pass. If a student, who gets 110 marks,
fails by 50 marks, find the maximum marks.

Sol: It is clear, Minimum Marks = 110 + 50 = 160
Let the maximum marks = x
So, 50% of x = 160
x = 320

17. In an examination, 40% students failed in Mathematics and 51% failed in Science. I f 16%
failed in both the subjects, find the percentage of those who passed in both the subjects.

Sol: Here, a = 40, b = 51 and c = 16
Percentage of students passing both the subjects
= (100 ( a + b - c ))%
= (100 (40+5116))%
= 25%.

AVERAGES

Average: The average of a number of quantities of the same kind is equal to their sum
divided by the number of those quantities. It is also called mean or arithmetic mean.
For example: The average of 1, 3, 5, 7 is
4
7 5 3 1 + + +
=
4
16
= 4
Basic Formulae:


1. Average =
quantities of Number
quantities of Sum

2. Sum of quantities = Average Number of quantities
3. Number of quantities =
Average
quantities of sum

4. If the number of quantities in two groups be a
1
and a
2
and their average is x and y,
respectively, then the combined average (average of all of them put together) is given by
2 1
2 1
a a
y a x a
+
+
.


5. If the average of a
1
quantities is x and the average of a
2
quantities out of them is y, the
average of remaining group (rest of the quantities) is given by
2 1
2 1
a a
y a x a

.
6. If x is the average of x
1
, x
2
,.., x
n
, then

a. The average of x
1
+ a, x
2
+

a,., x
n
+ a is x + a.

b. The average of x
1
- a, x
2
-

a,., x
n
- a is x - a.

c. The average of ax
1
, ax
2
,.., ax
n
is x a , where a 0.

d. The average of
a
x
., ,.........
a
x
,
a
x
n 2 1
is
a
x
.

7. The average of n quantities is equal to x. If one of the given quantities whose value is p,
is replaced by a new quantity having value q, the average becomes y,
then q = p + n (y x).

8. The average of n quantities is equal to x. If a quantity is removed, the average becomes
y. The value of the removed quantity is n (x y) + y.

9. The average of n quantities is equal to x. If a quantity is added, the average becomes y.
The value of the new quantity is n (y x) + y.
10. The average of first n natural numbers is
2
1 n +
.
11. The average of squares of natural numbers till n is
6
) 1 n 2 )( 1 n ( + +
.
12. The average of cubes of natural numbers till n is
4
) 1 n ( n
2
+
.

13. The average of odd numbers from 1 to n is
( )
2
1 + number odd last

14. The average of even numbers from 1 to n is
( )
2
2 + number even last

15. If n is odd: The average of n consecutive numbers, consecutive even numbers or
consecutive odd numbers is always the middle number.

16. If n is even: The average of n consecutive numbers, consecutive even numbers or
consecutive odd numbers is always the average of the middle two numbers.

17. The average of first n consecutive even numbers is (n + 1).

18. The average of first n consecutive odd numbers is n.

19. The average of squares of first n consecutive even numbers is
3
) 1 n 2 )( 1 n ( 2 + +
.
20. The average of squares of consecutive even numbers till n is
3
) 2 n )( 1 n ( + +
.
21. The average of squares of consecutive odd numbers till n is
3
) 2 n ( n +
.
22. If the average of n consecutive numbers is m, then the difference between the smallest
and the largest number is 2 (n 1).



23. Geometric Mean or Geometric Average: It is useful in calculating averages of ratios
such as average population growth rate, average percentage increase and so on.
Geometric mean of x
1
, x
2
,.., x
n
is denoted by
G.M. =
n
n 2 1
x . .......... x . x

24. Harmonic Mean or Harmonic Average: It is useful for finding average speed of a
vehicle, average production per day and so on.
H.M. =
(

+ +
n 2 1
x
1
..........
x
1
x
1
n
1
1

25. If a certain distance is covered at a speed of x km/h and the same distance is covered at
a speed of y km/h, the average speed during the whole journey is
(

+ y x
xy 2
km/h.
26. If a person or a motor car covers three equal distances at the speed of x km/h, y km/h, z
km/h, respectively, then for the entire journey average speed of the person or motor car
is
(

+ + zx yz xy
xyz 3
km/h.
27. If a person covers A km at a speed of x km/h, B km at a speed of y km/h and C km at a
speed of z km/h, then the average speed during the whole journey is
(
(
(
(

+ +
+ +
z
C
y
B
x
A
C B A
km/h.
28. If a person covers A
th
part of the distance at x km/h, B
th
part of the distance at y km/h
and the C
th
part at z km/h, then the average speed during the whole journey is
(
(
(
(

+ +
z
C
y
B
x
A
1
km/h.


SOLVED EXAMPLES

1. Sunil purchased 4 toys at the rate of Rs.100 each, 6 toys at the rate of Rs.150 each and 8
toys at the rate of 200 each. What is the average cost of one toy?

Sol: Cost of 4 toys = 100 x 4 = Rs.400
Cost of 6 toys = 150 x 6 = Rs.900
Cost of 8 toys = 200 x 8 = Rs.1600
Total number of toys = 4 + 6 + 8 = 18
Average price of 1 toy =
18
1600 + 900 + 400

= Rs.161.11

2. The average marks obtained by 100 students in a competitive examination is 50. Find the
total marks.

Sol: Total marks = Average marks x Number of students
= 100 x 50 = 5000.

3. The average weight of 40 students of section A of I-CET class is 55 kg and that of 50
students of section B is 60 kg. Find the average weight of all the 90 students of the class.



Sol: Here, a
1
= 40, a
2
= 50 and x = 55, y = 60
Average weight =
2 1
2 1
a a
y a x a
+
+

=
50 40
) 60 )( 50 ( ) 55 )( 40 (
+
+


= 57.78 kg.
4. Average salary of all the 30 employees including 5 officers of a company is Rs.7500. If the
average salary of the officers is Rs.15000, find the average salary of the remaining staff of
the company.

Sol: Here, a
1
= 30, a
2
= 5 and x = 7500, y = 15000
Average salary of the remaining staff =
2 1
2 1
a a
y a x a


=
5 30
) 1500 ( 5 ) 750 ( 30



= 6000.

5. The average value of five numbers 7, 10, 16, 24 and 28 is 17. If 6 is added to each number,
what will be the new average?
Sol: The new average = x + a = 17 + 6 = 23.

6. The average of x numbers is 4x. If x 3 is subtracted from each given number, what will be
the new average?
Sol: The new average = x - a = 4x (x 3) = 3x + 3.




7. The average of 8 numbers is 20. If each of the numbers is multiplied by 8, find the average
of a new set of numbers.
Sol: The average of a new set of numbers = a x = 8 x 20 = 160.

8. The average weight of 20 persons is increased by 3 kg when one of them whose weight is
50 kg, is replaced by a new person. What is the weight of the new person?
Sol: The weight of the new person, q = p + n (y x).
= 50 + 20(3)
= 50 + 60
= 110 kg.

9. The average age of 24 students and the Maths teacher is 16 years. If the Maths teachers
age is excluded, the average age reduces by 1 year. What is the age of the Maths teacher?

Sol: The age of Maths teacher = n ( x - y) + y
= 25 (16 15) + 15)
= 40 years.

10. Find the average of first 79 natural numbers.

Sol: The required average =
2
1 n +

=
2
1 79 +
= 40.
11. Find the average of squares of the natural numbers from 1 to 47.



Sol: The required average =
6
) 1 n 2 )( 1 n ( + +

=
| |
6
1 ) 47 ( 2 ) 1 47 ( + +

=
6
95 48
= 760.
12. Find the average of cubes of the natural numbers from 1 to 15.

Sol: The required average =
4
) 1 n ( n
2
+

=
4
) 1 15 ( 15
2
+

=
4
16 16 15
= 960.
13. Find the average of odd numbers from 1 to 50.

Sol: The required average =
2
1 number odd last +

=
2
1 49 +
= 25.
14. Find the average of even numbers from 1 to 61.

Sol: The required average =
2
2 number even last +

=
2
2 60 +
= 31.


15. Find the average of 5 consecutive numbers 4, 5, 6, 7, 8.

Sol: The required average = middle number = 6.

16. Find the average of consecutive odd numbers 21, 23, 25, 27, 29, 31.

Sol: The required average = average of middle two numbers
=
2
27 25 +
= 26.
17. Find the average of first 25 consecutive even numbers.

Sol: The required average = (n + 1) = 25 + 1 = 26.

18. Find the average of first 30 consecutive odd numbers.

Sol: The required average = n = 30.

19. Find the average of squares of first 16 consecutive even numbers.

Sol: The required average =
3
) 1 n 2 )( 1 n ( 2 + +

=
| |
3
1 ) 16 ( 2 ) 1 16 ( 2 + +

=
3
33 17 2
= 374.


20. Find the average of squares of consecutive even numbers from 1 to 35.

Sol: The required average =
3
) 2 n )( 1 n ( + +

=
3
) 2 + 34 )( 1 + 34 (

=
3
36 35
= 420
21. Find the average of squares of consecutive odd numbers from 1 to 44.
Sol: The required average =
3
) 2 + n ( n

=
3
) 2 + 43 ( 43

=
3
45 43
= 645
22. If the average of 6 consecutive numbers is 48, then find the difference between the
smallest and the largest number?

Sol: The required difference = 2 (n 1) = 2 (6 1) = 10.

23. The production of a company for three successive years has increased by 10%, 20% and
40% respectively. Find the average annual increase of production.
Sol: Average annual increase = G.M. of x, y and z =
3
1
) z y x ( %
=
3
1
) 40 20 10 ( % = 20%.



24. The population of a city in two successive years increases at the rates 12% and 3%
respectively. Find the average increase of two years.


Sol: Average population increase = G.M. of x and y
=
2
1
) y x ( %
=
2
1
) 3 12 ( %
= 6%.

25. A man runs 2 km at 20 km/ph and another 2 km at 10 km/ph. Find the average speed for
the whole distance in covering 4 km.
Sol: Required formula =
y + x
xy 2
and here, x = 20 and y = 10
So, Average speed =
(

+10 20
) 10 ( ) 20 ( 2
km/ph
=
3
40
=
3
1
13 km/ph.
26. A train covers the first 150 km at a speed of 100 km/ph, another 150 km at a speed of 120
km/ph and the last 150 km at 80 km/ph. Find the average speed of the train for the entire
journey.
Sol: Required Average speed =
(

+ + zx yz xy
xyz 3
km/ph


=
(

+ + ) 80 100 ( ) 80 120 ( ) 120 100 (


) 80 )( 120 )( 100 ( 3
km/ph
=
(

29600
2880000
km/ph
= 97.3 km/ph (app)


27. A person covers 8 km at a speed of 4 km/ph, 15 km at a speed of 5 km/ph and 20 km at a
speed of 10 km/ph. Find the average speed of the whole journey.
Sol: The required average speed =
(
(
(
(

+ +
+ +
z
C
y
B
x
A
C B A
km/ph.
=
10
20
+
5
15
+
4
8
20 + 15 + 8
km/ph.
=
(

+ +
+ +
2 3 2
20 15 8
km/ph.

=
7
1
6
7
43
= km/ph.
28. A person covers the first
th
4
1
of the distance at 5 km/ph, the next
th
5
3
at 8 km/ph and the
remaining distance at 10 km/ph. Find the average speed during the entire journey.
Sol: The required average speed =
z
C
+
y
B
+
x
A
1
km/ph
=
10
20 3
+
8
5 3
+
5
4 1
1
km/ph
=
200
3
+
40
3
+
20
1
1
km/ph
=
200
3 + 15 + 10
1
km/ph
=
28
200
km/ph =
7
1
7 km/ph.

29. A train covers 50% of the journey at 25 km/ph, 25% of the journey at 15 km/ph and the
remaining at 10 km/ph. Find the average speed during the entire journey.
Sol: The required average speed =
(
(
(
(

+ +
z
C
y
B
x
A
100
km/ph


=
(
(
(
(

+ +
10
25
15
25
25
50
100
km/ph

=
(
(
(
(

+ +
6
15 10 12
100
km/ph
=
37
8
16 =
37
600
km/ph.

RATIO AND PROPORTION

Ratio: A ratio is a comparison of two quantities by division.
If a and b are two numbers, then the ratio of a to b is
b
a
or a b. It is denoted by a : b.

Note:
1. The two quantities in a : b that are being compared are called terms.
2. The first term is called antecedent and the second term is called consequent.
Ex: The ratio 4:5 represents
5
4
with antecedent 4 and consequent 5.
3. A ratio is a number, so to find the ratio of two quantities; they must be expressed in the same
units.
4. A ratio does not change if both of its terms are multiplied or divided by the same number. Thus,
5
4
=
10
8
=
15
12
etc.


TYPES OF RATIOS:

1. Duplicate Ratio: If a : b is the given ratio then its duplicate ratio is a
2
: b
2
.
2. Triplicate Ratio: If a : b is the given ratio then its triplicate ratio is a
3
: b
3
.
3. Sub-duplicate Ratio: If a : b is the given ratio then its sub-duplicate ratio is a : b .
4. Sub-triplicate Ratio: If a : b is the given ratio then its sub-triplicate ratio is
3 3
b : a .
5. Inverse Ratio or Reciprocal Ratio: If a : b is the given ratio, then its inverse or reciprocal ratio is
b : a.
6. Compound Ratio: If a : b and c : d are two given ratios, then ac : bd is the compound ratio of
the given ratios.



Proportion: The equality of two ratios is called proportion.

Note:
1. If
b
a
=
d
c
, then a, b, c and d are said to be in proportion.
2. We write it as a : b :: c : d and read as a is to b as c is to d.
3. Each term of
b
a
and
d
c
is called a proportional.
4. In the a : b :: c : d proportion, a, d are known as extremes and b, c are known as means.



5. If four quantities are in proportion, then



Product of Means = Product of Extremes


Ex:
2
3
=
4
6
, we write as 3:2 :: 6:4 and say 3, 2, 6 and 4 are in proportion.
Here, 3 and 4 are extremes and their product = 3 x 4 = 12
2 and 6 are means and their product = 2 x 6 = 12

6. Fourth Proportional: If a : b :: c : x, then x is called the fourth proportional of a, b, c.
We have,
b
a
=
x
c
, or
a
c b
x

= .
Thus, fourth proportional of a, b, c is
a
c b
.
7. Third Proportional: If a : b :: b : x, then x is called the third proportional of a, b is b
2
/a


8. Mean Proportional: If a : b is the given ratio, then the mean proportional of a and b is ab .


Formulae:
1. If
b
a
=
d
c
, then
a) Componendo Rule:
b
b a +
=
d
d c +

b) Dividendo Rule:
b
b a
=
d
d c

c) Componendo and Dividendo Rule:
b a
b a

+
=
d c
d c

+

2. If two numbers are in the ratio of a : b and the sum of these numbers is x, then these
numbers will be
b a
ax
+
and
b a
bx
+
respectively.
3. If in a mixture of x litres two liquids A and B are in the ratio of a : b, then the quantities of
liquids A and B in the mixture will be
b a
ax
+
litres and
b a
bx
+
litres respectively.
4. If three numbers are in the ratio a : b : c and the sum of these numbers is x, then these
numbers will be
c b a
ax
+ +
,
c b a
bx
+ +
and
c b a
cx
+ +
respectively.
5. If two numbers are in the ratio a : b and the difference of these numbers is x, then these
numbers will be
a)
b a
ax

and
b a
bx

respectively, (where a > b).


b)
a b
ax

and
a b
bx

respectively, (where b > a).


6. The ratio between two numbers is a : b. If x is added to each of these numbers, the ratio
becomes c : d, then the two numbers are
bc ad
) d c ( ax

and
bc ad
) d c ( bx

.
7. The ratio between two numbers is a : b. If x is subtracted from each of these numbers, the
ratio becomes c : d, then the two numbers are
bc ad
) c d ( ax

and
bc ad
) c d ( bx

.
8. If the ratio of two numbers is a : b, then the number that should be added to each of the
numbers in order to make this ratio c : d is given by
d c
bc ad

.


9. If the ratio of two numbers is a : b, then the number that should be subtracted from each of
the numbers in order to make this ratio c : d is given by
d c
ad bc

.
10. There are four numbers a, b, c and d. The number that should be subtracted from each of the
numbers so that the remaining numbers may be proportional is given by
) c b ( ) d a (
bc ad
+ +

.
11. There are four numbers a, b, c and d. The number that should be added to each of the
numbers so that the new numbers may be proportional is given by
) c b ( ) d a (
ad bc
+ +

.
12. The incomes of two persons are in the ratio of a : b and their expenditures are in the ratio of
c : d. If the savings of each person be Rs.X, then their
Incomes are Rs.
bc ad
) c d ( aX

and Rs.
bc ad
) c d ( bX

and their
Expenditures are Rs.
bc ad
) a b ( cX

and Rs.
bc ad
) a b ( dX

.
13. If in a mixture of x litres of two liquids A and B, the ratio of liquids A and B is a : b, then the
quantity of liquid B to be added in order to make this ratio c : d is
) b a ( c
) bc ad ( x
+

.
14. In a mixture of two liquids A and B, the ratio of liquids A and B is a : b. If on adding x litres of
liquid B to the mixture, the ratio of A to B becomes a : c, then in the beginning the quantity of
liquid A in the mixture was
b c
ax

and that of liquid B was


b c
bx

litres.
15. When two ingredients A and B of quantities
1
q and
2
q and cost price per unit
1
c and
2
c are
mixed to get a mixture c having cost price per unit
m
c , then
a)
2
1
q
q
=
1 m
m 2
c c
c c

and
b)
2 1
2 2 1 1
m
q q
q c q c
c
+
+
=

16. If
1 1
d : n b : a = and
2 2
d : n c : b = , then

) d d ( : ) n d ( : ) n n ( c : b : a
2 1 2 1 2 1
=

17. If
1 1
: : d n b a = and
2 2
: : d n c b = and
3 3
: : d n d c = , then

) d d d ( : ) n d d ( : ) n n d ( : ) n n n ( d : c : b : a
3 2 1 3 2 1 3 2 1 3 2 1
=

18. If a mixture contains two ingredients A and B in the ratio a : b, then
a) Percentage of A in the mixture = % 100
b a
a

+
.
b) Percentage of B in the mixture = % 100
b a
b

+
.
19. If two mixtures
1
M and
2
M contain ingredients A and B in the ratios a : b and c : d
respectively, then a third mixture
3
M obtained by mixing
1
M and
2
M in the ratio x : y will
contain:
a)
(
(
(
(

+
+
+
+
% 100
y x
d c
cy
b a
ax
ingredient A, and


b)
(
(
(
(

+
+
+
+
% 100
y x
d c
dy
b a
bx
ingredient B or
(
(
(
(

+
+
+
+
% 100
y x
d c
cy
b a
ax
100 ingredient B





SOLVED EXAMPLES

1. The duplicate ratio of 4 : 5 is ______.

Sol: Duplicate ratio of 4 : 5 is
2 2
5 : 4 = 16 : 25.

2. The triplicate ratio of 2 : 3 is _______.

Sol: Triplicate ratio of 2 : 3 is
3 3
3 : 2 = 8 : 27.

3. The sub-duplicate ratio of 9 : 16 is ________.

Sol: Sub-duplicate ratio of 9 : 16 = 16 : 9 = 3 : 4.

4. The sub-triplicate ratio of 8 : 27 is _________.

Sol: Sub-triplicate ratio of 8 : 27 is
3 3
27 : 8 = 2 : 3.

5. The inverse ratio of 4 : 5 is _________.

Sol: Inverse ratio of 4 : 5 is 5 : 4.

6. The compound ratio of 3 : 4, 4 : 5 and 5 : 6 is _________.

Sol: Compound ratio of
3 : 4, 4 : 5 and 5 : 6 = (3 x 4 x 5) : (4 x 5 x 6)
= 60 : 120 = 1 : 2.

7. Find a fourth proportional to 2, 5, 6.

Sol: Let x be the fourth proportional to 2, 5, 6.
Then, 2 : 5 :: 6 : x or
x
6
=
5
2

So, x = 15.
8. Find a fourth proportional to 5, 4.

Sol: Let x be the third proportional to 5, 4.
Then 5 : 4 :: 4 : x or
x
4
4
5
=
2 . 3
5
16
x = = .


9. Find the mean proportional between 32 and 8.

Sol: The mean proportion between 32 and 8
= 16 256 8 32 = = .

10. Two numbers are in the ratio 3 : 4 and the sum of these numbers is 21.
Find the two numbers.

Sol: Here, a = 3, b = 4 and x = 21
The first number =
b a
ax
+
=
4 3
21 3
+

= 9
The second number =
b a
bx
+
=
4 3
21 4
+

= 12
(or)
= 21 9 = 12.

11. Two numbers are in the ratio 3 : 5 and the difference of these numbers is
36. Find the two numbers.

Sol: Here, a = 3, b = 5 and x = 36
The first number =
a b
ax

=
3 5
36 3

= 54
The second number =
a b
bx

=
3 5
36 5

= 90
(or)
= 54 + 36 = 90.

12. If a : b = 3 : 4 and b : c = 5 : 6, find a : b : c.

Sol: Here, 6 d , 4 d , 5 n , 3 n
2 1 2 1
= = = =
) d d ( : ) n d ( : ) n n ( c : b : a
2 1 2 1 2 1
=
) 6 4 ( : ) 5 4 ( : ) 5 3 ( =
24 : 20 : 15 = .

13. Given two numbers which are in the ratio of 2 : 3. If 6 is added to each of
them, their ratio is changed to 4 : 5. Find the two numbers.

Sol: Here, a : b = 2 : 3 and c : d = 4 : 5 and x = 6
The first number =
bc ad
) d c ( ax

=
) 4 ( 3 ) 5 ( 2
) 5 4 )( 6 ( 2



= 6
2
12
=


The second number =
bc ad
) d c ( bx

=
) 4 ( 3 ) 5 ( 2
) 5 4 )( 6 ( 3



= 9
2
18
=

.
14. The ratio of two numbers is 5: 9 If each number is decreased by 5, the
ratio becomes 5 : 11. Find the numbers.

Sol: Here, a : b = 5 : 9 and c : d = 5 : 11 and x = 5
The first number =
bc ad
) c d ( ax

=
) 5 ( 9 ) 11 ( 5
) 5 11 )( 5 ( 5



= 15
10
6 25
=




The second number =
bc ad
) c d ( bx

=
) 5 ( 9 ) 11 ( 5
) 5 11 )( 5 ( 9



= 27
10
6 45
=

.
15. Find the number that must be subtracted from the terms of the ratio 5 : 6
to make it equal to 2 : 3.

Sol: Here, a : b = 5 : 6 and c : d = 2 : 3
Required number =
d c
ad bc


= 3
3 2
) 3 ( 5 ) 2 ( 6
=

.
16. Find the number that must be added from the terms of the ratio 11 : 29 to
make it equal to 11 : 20.

Sol: Here, a : b = 11 : 29 and c : d = 11 : 20
Required number =
d c
bc ad

= 11
20 11
) 11 ( 29 ) 20 ( 11
=

.

17. Find the number subtracted from each of the numbers 54, 71, 75 and 99
leaves the remainders which are proportional.

Sol: Here, a = 54, b = 71, c = 75 and d = 99
Required number = 3
) 75 71 ( ) 99 54 (
) 75 ( 71 ) 99 ( 54
) c b ( ) d a (
bc ad
=
+ +

=
+ +

.

18. Annual income of Shashi and Ravi is in the ratio of 4 : 3 and the ratio of
their annual expenses is 3 : 2. If each of them saves Rs. 500 at the end of
year, then find their annual income.

Sol: Here, a : b = 4 : 3 and c : d = 3 : 2 and X = 500
So, Annual income of Shashi =
bc ad
) c d ( aX


=
1
2000
) 3 ( 3 ) 2 ( 4
) 3 2 )( 500 ( 4



= Rs.2000

Annual income of Ravi =
bc ad
) c d ( bX


=
1
1500
) 3 ( 3 ) 2 ( 4
) 3 2 )( 500 ( 3



= Rs.1500.

19. The incomes of Madhav and Kiran are in the ratio 9 : 4 and their
expenditures are in the ratio 5 : 2. If each saves Rs.1000, find their
expenditures.

Sol: Here, a : b = 9 : 4 and c : d = 5 : 2 and X = 1000
So, the expenditure of Madhav =
bc ad
) a b ( cX


=
2
5000
) 5 ( 4 ) 2 ( 9
) 5 4 )( 1000 ( 5



= Rs.2500
Expenditure of Kiran =
bc ad
) a b ( dX




=
2
2000
) 5 ( 4 ) 2 ( 9
) 5 4 )( 1000 ( 2



= Rs.1000.

20. 630 ml of a mixture contains milk and water in the ratio 7 : 2. How much
more water is to be added to get a new mixture containing milk and water
in the ratio of 7 : 3.

Sol: Here, a : b = 7 : 2 and c : d = 7 : 3 and x = 630
Required quantity of water to be added =
) b a ( c
) bc ad ( x
+


=
( ) 630 7(3) 2(7)
7(7 2)

+

=
630(21 14)
63

= 70 ml
21. A mixture contains milk and water in the ratio 8 : 3. On adding 6 litres of
water, the ratio of milk to water becomes 8 : 5. Find the quantity of water
in the mixture.

Sol: Here, a : b = 8 : 3 and a : c = 8 : 5 and x = 6
So, the quantity of water in the mixture =
b c
bx


= 9
3 5
) 6 ( 3
=

litres.
22. In what ratio the two kinds of tea powder (Ex: general tea powder with
lams a) must be mixed together into one at Rs.10 per kg and another at
Rs.15 per kg, so that the mixture may cost Rs.12.2 per kg?

Sol: Here, 10 c
1
= , 15 c
2
= , 2 . 12 c
m
=
Required ratio,
2
1
q
q
=
1 m
m 2
c c
c c


= 1 . 1 : 4 . 1
2 . 2
8 . 2
10 2 . 12
2 . 12 15
= =


Thus, the two kinds of tea powder are mixed in the ratio 1.4 : 1.1.



23. In a mixture of two types of liquids
1
L and
2
L , the ratio
2 1
L : L is 4 : 3. If
the cost of liquid
1
L is Rs.5 per litre and that of
2
L is Rs.10 per litre, then
find the cost per litre of the resulting mixture.

Sol: Here, 10 c , 5 c , 3 q , 4 q
2 1 2 1
= = = =
So, the cost of the resulting mixture,
2 1
2 2 1 1
m
q q
q c q c
c
+
+
=
=
2 1
2 2 1 1
q q
q c q c
+
+

=
7
50
3 4
3 10 4 5
=
+
+

= Rs.7.14 (app).

24. If a mixture contains water and alcohol in the ratio 3 : 7, what is the
percentage quantity of alcohol in the mixture?



Sol: Water (a) : alcohol (b) = 3 : 7
Percentage of alcohol in the mixture = % 100
b a
b

+

= % 100
7 3
7

+

= %. 70
Note: Similarly you can find the percentage of water.

25. Two alloys contain gold and copper in the ratio 3 : 2 and 5 : 2. In what
ratio the two alloys should be added together to get a new alloy having
gold and copper in the ratio of 2 : 1.

Sol: Here, a : b = 3 : 2 and c : d = 5 : 2
Let the two alloys be mixed in the ratio x : y.
Quantities of Gold and Copper should be added in the ratio
=
d + c
dy
+
b + a
bx
d + c
cy
+
b + a
ax

=
7
y 2
+
5
x 2
7
y 5
+
5
x 3

=
y 10 + x 14
y 25 + x 21

Since, the ratio of gold and copper in the new alloys is 2 : 1

y 10 + x 14
y 25 + x 21
=
2
1

21x+25y = 28x+20y
7x = 5y

x 5
y 7
=
Hence, the two alloys should be mixed in the ratio 5 : 7.






PARTNERSHIP

Partnership: Two or more persons carry on a business and share the profits of the business at
an agreed proportion. This is called partnership.

Partners: Persons who have entered into partnership with one another are individually called
partners. They are of two types.

a) Sleeping Partner: A person who invests the capital in the business but does not actively
participate in the conduct of business is called sleeping partner.
b) Working Partner: A person who takes part in running the business besides investing the
capital is called working partner. He gets salary for his work or some per cent of profit, in
addition.

Firm name: The name under which the business is carried on is called firm name.



Note: The partnership may be simple or compound.

Simple Partnership: It is one in which the capital of each partner is in the business for same
time.

Compound Partnership: It is one in which the capitals of partners are invested for different
periods.

Results:

1. Three partners A, B and C invested their capitals in a business in the ratio of
3 2 1
C : C : C
and their profits are in the ratio
3 2 1
P : P : P , then the ratio of timing of their investments is
3
3
2
2
1
1
C
P
:
C
P
:
C
P
.

2. Three partners A, B and C invested their capitals in a business. If the timing of their
investments is in the ratio
3 2 1
t : t : t and their profits are in the ratio
3 2 1
P : P : P , then the
ratio of their capitals invested is
3
3
2
2
1
1
t
P
:
t
P
:
t
P
.

3. If capitals of two partners A, B be C be Rs.
1
C and Rs.
2
C respectively for the periods
1
t ,
2
t and
3
t respectively, then
Profit of A : Profit of B : Profit of C =
2 2 1 1
t C : t C :
3 3
t C
4. If there is a loss in the business, then
Loss of A : Loss of B : Loss of C =
2 2 1 1
t C : t C :
3 3
t C
(Note: Investment Ratio = Profit Ratio)


SOLVED EXAMPLES

1. Shashi, Ravi and Kiran invested Rs.2000, Rs.5000 and Rs.4000 respectively in a business.
The net profit for the year was Rs.1210 which was divided in proportion to investments. Find
the profit of each.

Sol: Here, 2000 = C
1
, 5000 C
2
= and 4000 C
3
= and P = 1210
Ratio of capitals = 2:5:4
Shashis profit =
2
2 5 4 + +
x 1210 = 220
Ravis profit =
5
11
x 1210 = 550
Kirans profit =
4
11
x 1210 = 440

Note: You can solve in this way also.
Profit of Kiran = Total Profit - (Profit of Shashi + Profit of Ravi)

2. A and B are two partners in a business. A contributes Rs. 1000 for 6 months and B Rs. 600
for 5 months. If total profit is Rs.600, find the profits of A and B.

Sol: Here, 1000 C
1
= , 600 C
2
= and 6 t
1
= , 5 t
2
= and P = 600



Profit of A : Profit of B = C
1
t
1
: C
2
t
2

= 6000 : 3000
= 2 : 1
Profit of A =
2
3
x 600 = 400
Profit of B =
1
3
x 600 = 200

3. A, B and C are three partners in a business. A contributes Rs.1500 for 6 months and B
Rs.1200 for 4 months and C Rs.2000 for 2 months. Find the ratio of their shares in the
profit.

Sol: Here, 1500 C
1
= , 1200 C
2
= and 2000 C
3
= and
6 t
1
= , 4 t
2
= and 2 t
3
=
Profit of A : Profit of B : Profit of C =
2 2 1 1
t C : t C :
3 3
t C
= 1500(6) : 1200(4) : 2000(2)
= 9000 : 4800 : 4000
= 45 : 24 : 20.

4. Raju, Suman and Sunil invested capitals in the ratio of 3 : 4 : 7. At the end of the business
term, their profits are in the ratio 1 : 3 : 5. Find the ratio of time for which they invested
the capitals.

Sol: Here, 7 : 4 : 3 = C : C : C
3 2 1
and 5 : 3 : 1 P : P : P
3 2 1
=
Required ratio = 60 : 63 : 28
7
5
:
4
3
:
3
1
C
P
:
C
P
:
C
P
3
3
2
2
1
1
= = .
5. Phani, Deepa and Bindu start a business. If the ratio of their periods of investments are 1 :
4 : 5 and their profits are in the ratio 4 : 8 : 10, find the ratio of their capitals.

Sol: 10 : 8 : 4 P : P : P
3 2 1
= and 5 : 4 : 1 = t : t : t
3 2 1

Required ratio = 1 : 1 : 2 2 : 2 : 4
5
10
:
4
8
:
1
4
t
P
:
t
P
:
t
P
3
3
2
2
1
1
= = = .
MIXTURES (OR) ALLIGATION

Alligation: It means linking.

a) It is a rule to find the ratio in which two or more ingredients at their respective prices
should be mixed to give a mixture at a given price.

b) It is a rule to find the average price of a mixture when the prices of two or more
ingredients which may be mixed together and the proportion in which they are mixed
are given.

Mean Price: The cost price of a unit quantity of mixture is called the mean price.

Alligation Rule:

1. Let us suppose Rs.a per unit be the price of the first ingredient (superior quality) is
mixed with another ingredient (cheaper quality) of price Rs.b per unit to form a mixture
whose mean price is Rs.m per unit, then the two ingredients must be mixed in the ratio:

b m
m a
cheaper C.P.of - price Mean
price Mean - superior of C.P.
superior of Quantity
cheaper of Quantity

= =



2. This means, the two ingredients are to be mixed in the inverse ratio of the differences of
their prices and the mean price.

Note: This can be remembered easily through the diagram below:



Formulae:

1. A vessel contains a litres of wine. From it b litres are withdrawn. The vessel is then filled
with water. Next b litres of the mixture are withdrawn and again the vessel is filled with
water. If this process is repeated n times then


n
a
b a
vessel the in wine of quantity Original
operation nth after vessel the in left Wine
|
|
.
|

\
|
=
After n
th
the quantity of wine left in the vessel =
n
b
a 1
a
(
| |
(

|
(
\ .



2. There are n vessels of equal size filled with mixtures of liquids A and B in the ratio a
1
:
b
1
, a
2
: b
2
,,a
n
: b
n
, respectively. If the contents of all the vessels are poured into a
single large vessel, then

|
|
.
|

\
|
+
+ +
+
+
+
|
|
.
|

\
|
+
+ +
+
+
+
=
n n
n
2 2
2
1 1
1
n n
n
2 2
2
1 1
1
b a
b
...... ..........
b a
b
b a
b
b a
a
...... ..........
b a
a
b a
a
B liquid of Quantity
A liquid of Quantity


3. There are n vessels of sizes c
1
, c
2
, .. , c
n
filled with mixtures of liquids A and B in the
ratio a
1
: b
1
, a
2
: b
2
,,a
n
: b
n
, respectively. If the contents of all the vessels are
poured into a single large vessel, then

|
|
.
|

\
|
+
+ +
+
+
+
|
|
.
|

\
|
+
+ +
+
+
+
=
n n
n n
2 2
2 2
1 1
1 1
n n
n n
2 2
2 2
1 1
1 1
b a
c b
...... ..........
b a
c b
b a
c b
b a
c a
...... ..........
b a
c a
b a
c a
B liquid of Quantity
A liquid of Quantity


SOLVED EXAMPLES

1. In what ratio the two varieties of coffee one costing Rs.30 per kg and the other Rs.35
per kg should be blended to produce a blended variety of coffee worth Rs.32 per kg.
How much should be the quantity of second variety of coffee, if the first variety is 72
kg.


Sol:










The required ratio of the two
varieties of coffee is 3 : 2. i.e.

2
3
quality superior of Quantity
quality cheaper of Quantity
=
So, Quantity of superior coffee = 48
3
2
72 = kg.

2. Salt at Rs.10 per kg is mixed with salt at Rs.15 per kg in the ratio 3 : 4. Find the price
per kg of mixture.

Sol: Let the mean price of the mixture be Rs.x.




.
7
6
12 . Rs
7
90
x 90 x 7 x 4 60 30 x 3
10 x
x 15
4
3
10 x
x 15
salt superior of Quantity
salt cheaper of Quantity
= = = =

=


3. A vessel contains 100 litres of wine. 20 litres of wine was taken out and replaced by
water. Then, 20 litres of mixture was withdrawn and again replaced by water. The
operation was repeated for fourth time. How much wine is now left in the vessel?

Sol: Amount of wine left in the vessel = . litres 96 . 40 100
5
4
100
100
20
1
4 4
=
|
|
.
|

\
|
=
|
|
.
|

\
|

4. Two equal glasses are filled with mixture of milk and water. The proportion of milk and
water in the first glass is 4 : 3 and in the second glass is 5 : 2. The contents of the
two glasses are emptied into a single vessel. What is the proportion of milk and water
in it?

Sol: . 5 : 9
5
9
2 5
2
3 4
3
2 5
5
3 4
4
water of Quantity
milk of Quantity
= =
+
+
+
+
+
+
=



5. Three glasses of sizes 1 litres, 2 litres and 3 litres contain mixture of milk and water in
the ratio 2: 3, 3 : 7 and 4 : 11 respectively. The contents of all the three glasses are
poured into a single vessel. Find the ratio of milk to water in the resulting mixture.
Sol: =
water of Quantity
milk of Quantity
126
54
30
66 42 18
30
24 18 12
15
33
10
14
5
3
15
12
10
6
5
2
11 4
3 11
7 3
2 7
3 2
1 3
11 4
3 4
7 3
2 3
3 2
1 2
=
+ +
+ +
=
|
.
|

\
|
+ +
|
.
|

\
|
+ +
=
|
.
|

\
|
+

+
+

+
+

|
.
|

\
|
+

+
+

+
+


= 54 : 126 or 27 : 63.

PROFIT AND LOSS

1. In any business transaction, it is common to have either profit or loss.
2. But the aim of any business is to earn profit.
3. The most commonly used term involving sale and purchase of any business are Cost Price and
Selling Price.

Cost Price: The price at which an article has been purchased is called the cost price. It is denoted
by C.P.

Selling Price: The price at which an article has been sold is called the selling price. It is denoted
by S.P.

Profit or Gain: If S.P. > C.P., then there is a gain or profit.
Thus, Profit or Gain = S.P. C.P.

Loss: If C.P. > S.P., then there is a gain or profit.
Thus, Loss = C.P. S.P.

Note: Profit and Loss are always calculated w.r.t. C.P. of the item.

Formulae:

1. Gain % =
C.P.
100 Gain

2. Loss % =
C.P.
100 Loss

3. When S.P. and gain% are given, then
S.P.
Gain% 100
100
C.P.
|
|
.
|

\
|
+
=
4. When C.P. and gain% are given, then
C.P.
100
Gain% 100
S.P.
|
|
.
|

\
| +
=
5. When S.P. and loss% are given, then
S.P.
Loss% 100
100
C.P.
|
|
.
|

\
|

=
6. When C.P. and loss% are given, then
C.P.
100
Loss% 100
S.P.
|
|
.
|

\
|
=
7. If a man buys a items for Rs.b and sells c items for Rs.d, then the
Gain or loss % =
ad bc
100
bc






Note: a) In case of gain percent, the result will be positive.
b) In case of loss percent, the result will be negative.

8. If the C.P. of x articles = S.P. of y articles, then
Gain or loss % = % 100
y
y x

|
|
.
|

\
|

Note: a) If x > y, it is % gain.
b) If x < y, it is % loss.

9. The Cost Price of an article is C.P. If it is sold at S.P
1
., then gain% or loss% is x and if it is sold
at a price S.P
2
., gain% or loss% is y then
y 100
S.P
x 100
S.P
2 1
+
=
+
(Or)
y x
S.P S.P
100
C.P
2 1

=

Note: a) If x or y is negative it indicates a loss.
b) If x or y is positive it indicates a gain.

10. A sells an article to B at a gain or loss of x%, and B sells it to C at a gain or loss of y%. If C pays
Rs.z for it to B, then
C.P. for A =
(
(

+ +

) y 100 )( x 100 (
z 100
2

Note: a) If x or y is negative it indicates a loss.
b) If x or y is positive it indicates a gain.

11. If A sells an article to B at a gain or loss of x%, and B sells it to C at a gain or loss of y% then
the
Resultant Profit% or Loss % =
|
|
.
|

\
|
+ +
100
xy
y x
Note:
1. If x or y is negative it indicates a loss.
2. If x or y is positive it indicates a gain.
3. This expression represents resultant profit% or loss% according as it is positive or
negative.

12. If two different articles are sold at the same S.P., getting gain or loss of x% on the first
transaction and gain or loss of y% on the second transaction, then the
Overall % gain or % loss = %
) y 100 ( ) x 100 (
xy 2 ) y x ( 100
|
|
.
|

\
|
+ + +
+ +

Note: This expression represents overall gain or loss according as its sign is positive or
negative.]

13. If two different articles are sold at the same S.P., getting gain of x% on the first transaction and
gain or loss of x% on the second transaction, then the
Overall % loss = %
10
x
2
|
|
.
|

\
|

Note: In this type of questions there is always a loss.

14. If a shopkeeper uses faulty measure and sells his goods at a gain or loss of x% then the
Overall % gain or % loss is
measure Faulty
measure True
x 100
g 100
=
+
+

Note: If the merchant sells his goods at C.P., then x = 0.





15. If a merchant uses y% less weight or length and sells his goods at a gain or loss of x% then the
Overall % gain or loss = % 100
y 100
x y
(
(

|
|
.
|

\
|

+


16. If a person buys two items for Rs.A and sells one at a loss of l% and other at a gain of g% and if
each item was sold at the same price, then
a. The C.P. of the item sold at loss =
gain%) (100 loss%) - (100
gain%) 100 ( A
+ +
+

b. The C.P. of the item sold at gain =
gain%) (100 loss%) - (100
loss%) 100 ( A
+ +



17. If two successive discounts on an article at x% and y% respectively, then the
Overall Discount = %
100
xy
y x
|
|
.
|

\
|
+
18. If three successive discounts on an article at x%, y% and z% respectively, then the
Overall Discount = %
100
xyz
100
) zx yz xy (
z y x
2
|
|
.
|

\
|
+
+ +
+ +
19. A shopkeeper sells an item at Rs.x after giving a discount of d% on labeled price. Had he not
given the discount, he would have earned a profit of p% on the C.P. then the C.P. of each item is
given by
C.P. =
|
|
.
|

\
|
+ ) p 100 )( d 100 (
x 100
2


SOLVED EXAMPLES

1. If C.P. = Rs.440, S.P. = Rs.480 then find the profit.

Sol: C.P. = Rs.440, S.P. = Rs.480
Profit = S.P. C.P. = 480 440 = Rs.40.

2. If C.P. = Rs.135, S.P. = Rs.120 then find the loss.

Sol: C.P. = Rs.135, S.P. = Rs.120
Loss = C.P. S.P. = 135 120 = Rs.15.

3. The cost price of a pen is Rs.400 and the selling price is Rs.460. Find the gain %.

Sol: C.P. = Rs.400, S.P. = Rs.460
Gain = S.P. C.P. = 460 400 = Rs.60
Gain % = 15% 100
400
60
C.P.
100 Gain
= =

.

4. Rajani bought a jewel for Rs.825 and sold it for Rs.750. Find the loss%.

Sol: C.P. = Rs.825, S.P. = Rs.750
Loss = C.P. S.P. = 825 750 = Rs.75
Loss % = %
11
1
9
11
100
825
100 75
C.P.
100 Loss
= =

.

5. Shashi buys a T.V. set for Rs.9500. For how much should he sell in order to gain 4%?

Sol: C.P. = Rs.9500, gain% = 4%


C.P.
100
Gain% 100
S.P.
|
|
.
|

\
| +
= , if C.P. and gain% are given
9880 Rs. 95 104 500 9
100
4 100
S.P. = =
|
|
.
|

\
| +
= .
6. Hari loses 8% by selling a Cooler for Rs.5520. Find the C.P. of the cooler.

Sol: S.P. = Rs.5520, loss% = 8%
S.P.
Loss% 100
100
C.P.
|
|
.
|

\
|

= , if S.P. and loss% are given


6000 Rs. 5520
92
100
520 5
8 100
100
C.P. = =
|
|
.
|

\
|

= .
7. By selling a toy for Rs.422, Kiran gains %
2
1
5 . Find the C.P. of the toy.
Sol: S.P. = Rs.422, gain% = %
2
11
= %
2
1
5 .
S.P.
Gain% 100
100
C.P.
|
|
.
|

\
|
+
= , if S.P. and gain% are given
400 Rs. 422
211
200
422
2
1 1
100
100
C.P. = =
|
|
|
|
.
|

\
|
+
=
8. Venu buys oranges at the rate of Rs.10 per dozen and sells them at rate of 16 for Rs.11. Find
his % gain or loss.

Sol: a = 12, b = Rs.10, c = 16, d = Rs.11
Gain or loss% = % 100 1
160
132
% 100 1
bc
ad

|
|
.
|

\
|
=
|
|
.
|

\
|

% 5 . 17 % 100
160
28
= =
Since, the result is negative; there is a loss of 17.5%

9. 11 apples are bought at Rs.10 and sold at 10 for Rs.11. What is gain or loss%?

Sol: a = 11, b = Rs.10, c = 10, d = Rs.11

Gain or loss% = % 100 1
100
121
% 100 1
bc
ad

|
|
.
|

\
|
=
|
|
.
|

\
|

= %. 21 % 100
100
21
=

10. The selling price of 18 articles is equal to the cost price of 10 articles. What is the profit%?

Sol: x = 18, y = 10
Gain% = %. 80 % 100
10
10 18
% 100
y
y x
=

=
|
|
.
|

\
|


11. By selling a mobile for Rs.2400, Phani lost 10%. What percent shall he gain or lose by selling
it for Rs.2800.



Sol:S.P
1
.= Rs.2400, x = - 10%, S.P
2
.= Rs.2800, y =?
Here negative sign for x indicates loss.
We have,
y 100
S.P
x 100
S.P
2 1
+
=
+


5 y 30 y 6
630 y 6 600
y 100
7
90
6
y 100
2800
10 100
2400
= =
= +
+
=
+
=

Since the result is positive, Phani can have a gain of 5% by selling it for Rs.2800.

12. Avinash sells a bike to Shireesh at a gain of 15% and Shireesh again sells to Teja at a profit of
10%. If Teja pays Rs.37950 to Shireesh then what is the cost price of the bike for Avinash?

Sol: Here, x = 15, y = 10 and z = Rs.37,950
C.P. for Avinash =
(
(

+ +

) y 100 )( x 100 (
z 100
2


. 000 , 30 Rs. =
110 115
37950 100 100
=
) 10 + 100 )( 15 + 100 (
37950 100 100
=


13. Mounika sells an i-pod to Bindu at a gain of 12% and Bindu again sells it to swapna at a loss
of 20%. If swapna pays Rs.2240 to Bindu then what is the cost price of i-pod to Mounika?

Sol: Here, x = 12, y = - 20 and z = Rs.2240
C.P. for Mounika =
(
(

+ +

) y 100 )( x 100 (
z 100
2


. 2500 Rs.
4
100 100
80 112
2240 100 100
) 20 100 )( 12 100 (
2240 100 100
=


=
(

+

=

14. Sachin sells a machine to Krishna at a profit of 3% and Krishna sells it to Dhruva at a profit of
5%. Find the resultant profit%.

Sol: Here, x = 3, y = 5
Resultant Profit% =
|
|
.
|

\
|
+ +
100
xy
y x
= %
20
3
8
100
15
8
100
) 5 ( 3
5 3 = + =
|
|
.
|

\
|
+ + .
15. Pratap sells a trouser to Rajesh at a profit of 10% and Rajesh sells it to Chandu at a loss of
8%. Find the resultant profit or loss%.

Sol: Here, x = 10, y = - 8
Resultant Profit or loss% =
|
|
.
|

\
|
+ +
100
xy
y x

%.
5
1
1
100
120
100
80
2
100
) 8 ( 10
) 8 ( 10
= =
=
|
|
.
|

\
|
+ + =

This represents profit as the sign is positive.

16. Manoj sold two laptops, each for Rs.30000. If he makes 15% profit on the first and 10% loss
on the second, what is his gain or loss% on the whole transaction?



Sol: Here, x = 15, y = - 10
Overall % gain or % loss = %
) y 100 ( ) x 100 (
xy 2 ) y x ( 100
|
|
.
|

\
|
+ + +
+ +

= %
) 10 100 ( ) 15 100 (
) 10 )( 15 ( 2 ) 10 15 ( 100
|
|
.
|

\
|
+ +
+

= %.
41
40
%
205
200
%
205
300 500
= =
|
|
.
|

\
|

This represents profit as the sign is positive.

17. Chary sold two bicycles for Rs.1000 each, gaining 20% on one and losing 20% on the other.
Find his total gain or loss%.

Sol: Here, x = 20
Overall % loss = %
10
x
2
|
|
.
|

\
|

= %. 4 = %
10
20
2

18. A dishonest shopkeeper professes to sell his goods at C.P. but uses a 950 g for kg weight.
Find his gain%.

Sol: Here, True measure = 1000 g, False measure = 950 g, x = 0
Overall gain% is given by

measure Faulty
measure True
x 100
g 100
=
+
+

%.
19
5
5
19
100
g
100 g 19 2000 g 19 1900
19
20
100
g 100
950
1000
100
g 100
= =
= = +
=
+
=
+



19. A shopkeeper sells the goods at 22% loss on the C.P. but uses 10% less weight. What is his
gain or loss%?

Sol: Here, x = - 22, y = 10
Overall gain or loss% = % 100
10 100
22 10
(

|
|
.
|

\
|


=
3
40
9
120
% 100
90
12
= =
(


= %.
3
1
13
This represents loss as the sign is negative.

20. Suresh buys two books for Rs.615 and sells one at a loss of 10% and other at a gain of 15%.
If both the books are sold at the same price, then find the cost price of two books.

Sol: The C.P. of the item sold at loss of 10%
=
gain%) (100 loss%) - (100
gain%) 100 ( A
+ +
+

=
205
115 615
15) (100 10) - (100
15) 100 ( 615
=
+ +
+

= 3 x 115 = Rs.345.



The C.P. of the item sold at gain of 15%
=
gain%) (100 loss%) - (100
loss%) 100 ( A
+ +


=
205
90 615
15) (100 10) - (100
10) 100 ( 615
=
+ +


= 3 x 90 = Rs.270.

21. Find a single discount equivalent to two successive discounts of 20% and 30%.

Sol: Overall Discount = %
100
xy
y x
|
|
.
|

\
|
+
= %. 44 %
100
) 30 ( 20
30 20 =
|
|
.
|

\
|
+
22. Find a single discount equivalent to two successive discounts of 20% and 30% and 40%.

Sol: Overall Discount
= %
100
xyz
100
) zx yz xy (
z y x
2
|
|
.
|

\
|
+
+ +
+ +
= %
100
) 40 )( 30 )( 20 (
100
) 20 )( 40 ( ) 40 )( 30 ( ) 30 )( 20 (
40 30 20
2
|
|
.
|

\
|
+
+ +
+ +
= %
10000
24000
100
2600
90 %
100 100
24000
100
800 1200 600
90
|
|
.
|

\
|
+ =
|
|
.
|

\
|

+
+ +

= 66.4%. )% 4 . 2 26 90 ( = +

23. A shopkeeper sold the shirts at Rs.306 each after giving 15% discount on labelled price. If he
had not given the discount, he would have earned a profit of 20% on the cost price. Find the
C.P. of each shirt.

Sol: Here, labeled price x = Rs.306, discount d = 15%, profit p = 20%
Required C.P. =
|
|
.
|

\
|
+ ) p 100 )( d 100 (
x 100
2

=
120 85
306 100 100
) 20 100 )( 15 100 (
306 100 100


=
+


= Rs.300.

PROBLEMS ON AGES

In solving the problems related to ages, we come across three situations.
1. Age some years ago
2. Present Age
3. Age some years hence

Models:

1. If the age of A, t years ago, was n
1
times the age of B and at present the age of A is n
2
times
that of B, then finding the present Age of A and the present Age of B.

2. If the present age of A is n
1
times the present age of B. If t years hence, the age of A would
be n
2
times that of B, then finding the present Age of A the present Age of B.



3. If the age of A, t
1
years ago, was n
1
times the age of B and if t
2
years hence As age would be
n
2
times that of B, then finding the present Age of A and the present Age of B.

4. The sum of present ages of A and B is S years. If t years ago, the age of A was n times the
age of B, then finding the present Age of A and the present Age of B.

5. The sum of present ages of A and B is S years. If t years hence, the age of A would be n
times the age of B, then finding the present Age of A and the present Age of B.

6. If the ratio of the present ages of A and B is a : b and t years hence, it will be c : d, then
finding the present Age of A and the present Age of B.

SOLVED EXAMPLES

1. The age of Mr. Bindra is 3 times the age of his son. If 10 years ago, his age was 8 times
the age of his son, then at that time what would be the age of Mr. Bindra.

Sol: 10 years ago: present:
Son = x son = x+10
Mr. Bindra = 8x Mr. Bindra = 8x+10
3(x+10) = 8x+10
3x+30 = 8x+10
x = 4
Mr. Bindra = 8x4+10 = 42

2. The age of Mr. Sinha is 7 times the age of his daughter. After 15 years, the age of Mr.
Sinha is only twice the age of his daughter. Find the present age of Mr. Sinhas daughter.

Sol: Present: After 15 years:
Daughter = x Daughter = x+15
Mr. Sinha = 7x Mr. Sinha = 7x+15
7x+15 = 2(x+15)
7x+15 = 2x+30
x = 3
Mr. Sinhas daughter = 3 years

3. 12 years ago Lekhas father was 5 times older than her. After 12 years, the father will be 3
times older than her. Find the present age of Lekha.



Sol: 12 years ago: Present: After 12 years:
Lekha = x Lekha = x+12 Lekha = x+24
Father = 5x Father = 5x+12 Father = 5x+24
3(x+24) = 5x+24
3x+72 = 5x+24
x = 24 Lekha = 24+12 = 36
4. The sum of the ages of A and B is 36 years. 2 years back, the age of A is 4 times the age of
B. Find the difference between the present ages of A and B.

Sol: 2 years back: Present:
B = x B = x+2
A = 4x A = 4x+2
x+2+4x+2 = 36
x = 6.4
A-B = 25.6 6.4 = 19.2



5. The sum of the ages of father and a son is 44 years. After 8 years, the age of the father
will be 5 times that of son. Find their respective ages.

Sol: After 8 years: Present:
Son = x Son = x 8
Father = 5x Father = 5x-8
x-8+5x-8 = 44
x = 10
Son = 10-8 = 2
Father = 50-8 = 42

6. The ratio of the present ages of father and son is 3 : 1. After 10 years, it will become 5 : 2.
Find the present age of the son.

Sol: Present: After 10 years:
Son = x Son = x+10
Father = 3x Father = 3x+10

3x 10 5
x 10 2
+
=
+

6x+20 = 5x+50
x = 30
son = 30 years

7. 4 years ago Kalyani was thrice as old as Ramya. If the ratio of their present ages is 4 : 3
respectively. Find the difference between their ages.

Sol: Present: 4 years ago:
Kalyani = 4x Kalyani = 4x-4
Ramya = 3x Ramya = 3x-4
3(3x-4) = 4x-4
9x-12 = 4x-4
x = 1.6
Kalyani Ramya = 1.6



TIME AND WORK

Formulae:

1. If A can do a piece of work in n days, then at a uniform rate
of working, A will finish th
n
1
work in one day.
2. If th
n
1
of work is done by A in one day, then A will take n
days to complete the full work.

3. If A does th
n
1
of a work in m hours, then A will take
m
n
hours.

4. If A does three times faster work than B, then

a) The ratio of work done by A and B is 3 : 1

b) The ratio of time taken by A and B is 1 : 3

5. A, B and C can do a piece of work in T
1
, T
2
and T
3
days
respectively. If they have worked for D
1
, D
2
and D
3
days
respectively, then
a) Amount of work done by A =
1
1
T
D

b) Amount of work done by B =
2
2
T
D

c) Amount of work done by C =
3
3
T
D

d) Amount of work done by A, B and C together = .
T
D
+
T
D
+
T
D
3
3
2
2
1
1

6. If A can do a piece of work in X days and B can do the same
work in Y days, then both of them working together will do
the same work in
Y + X
XY
days.

7. If A, B and C, while working alone, can complete a work in
X, Y and Z days respectively, then they will together
complete the work in
ZX + YZ + XY
XYZ
days.

8. A and B working together can complete a piece of work in X
days. If A working alone can complete the work in Y days,
then B working alone will complete the work in
X Y
XY
-
days.



9. If A and B working together can complete a piece of work in
X days, B and C in Y days, C and A in Z days, then
a) A, B and C working together will finish the job in
ZX YZ XY
XYZ 2
+ +
days.
b) A alone can finish the job in
ZX YZ XY
XYZ 2
+
days.
c) B alone can finish the job in
XY ZX YZ
XYZ 2
+
days.
d) C alone can finish the job in
ZX YZ XY
XYZ 2
+
days.

10. If A and B working together can complete a work in X
days and B is k times efficient than A, then

a) The time taken by A working alone, to complete the
work is (k + 1) X.
b) The time taken by B working alone, to complete the
work is . x
k
1 k
|
|
.
|

\
| +

11. If A working alone takes a days more than A and B
working together. If B worked alone, he takes b days more
to complete the job than A and B working together then
both A and B working together can finish the job in ab
days.
12. If A can complete
b
a
part of a work in X days, then
d
c
part of the work will be done in
d a
X c b


days.

13. If a men and b women can do a piece of work in n
days, then c men and d women can do the work in
|
|
.
|

\
|
+ ad bc
nab
days.

14. If A can complete a work in X days and B is k times
efficient than A, then the time taken by both A and B
working together to complete the work is .
k 1
x
+



15. If A is k times more efficient than B and hence able to
finish the work in l days less than B, then
a) A working alone can finish the work in
1 k
l

days.
b) B working alone can finish the work in
1 k
kl

days.


c) A and B working together, can finish the work in
1 k
kl
2

days.

16. There are two groups of people with same efficiency.
In one M
1
persons can do W
1
works in D
1
time and in the
other M
2
persons can do W
2
works in D
2
time. Then the
relationship between the two groups is
1 2 2 2 1 1
W D M W D M =

17. There are two groups of people with same efficiency.
In one M
1
persons can do W
1
works in D
1
time working t
1

hours a day and in the other M
2
persons can do W
2
works in
D
2
time working t
2
hours a day. Then the relationship
between the two groups is
1 2 2 2 2 1 1 1
W t D M W t D M =

SOLVED EXAMPLES

1. A can complete a piece of work by working alone in 5 days and B while working alone
can finish the same work in 10 days. If both of them work together, then in how many
days, the work will be finished.

Sol: Here, X = 5, Y = 10
Working together, A and B will finish the work in
Y X
XY
+
days
=
3
10
10 5
) 10 ( 5
=
+
days.
2. A, B and C can complete a piece of work in 5, 10 and 13 days respectively. In how
many days would all of them complete the same work working together?

Sol: Here, X = 5, Y = 10, Z = 13
So, the work will be completed in
ZX YZ XY
XYZ
+ +
days
=
49
32
2
) 5 ( 13 ) 13 ( 10 ) 10 ( 5
13 10 5
=
+ +

days.

3. A and B working together take 10 days to complete a piece of work. If A alone can do
this work in 15 days, how long would B take to complete the same work?

Sol: Here, X = 10, Y = 15
So, B alone will complete the work in
X Y
XY

days = 30
10 15
) 15 ( 10
=

days.

4. Madhu and Anil can do a piece of work in 12 days, Ani l and Sunil in 15 days, Sunil and
Madhu in 20 days. How long would each take separately to do the same work?

Sol: Here, X = 12, Y = 15 and Z = 20
Madhu alone can do the work in
ZX YZ XY
XYZ 2
+
days
= 30
240
7200
) 12 ( 20 ) 20 ( 15 ) 15 ( 12
20 15 12 2
= =
+

days.


Anil alone can do the work in
XY ZX YZ
XYZ 2
+
days
= 20
360
7200
) 15 ( 12 ) 12 ( 20 ) 20 ( 15
20 15 12 2
= =
+

days.
Sunil alone can do the work in
ZX YZ XY
XYZ 2
+
days
= 60
120
7200
) 12 ( 20 ) 20 ( 15 ) 15 ( 12
20 15 12 2
= =
+

days.

5. Mukesh can do a piece of work in 16 days. If Nita works thrice as fast as Mukesh, how
long would they take to finish the work by working together?

Sol: Here, X = 16 and k = 3
Required time =
|
|
.
|

\
|
+ k 1
x
days
= 4
3 1
16
=
|
|
.
|

\
|
+
days.
1. A and B together can do a piece of work in 4 days. If A does twice as much work as B
in a given time, how long A alone would take to do the work?

Sol: Here, X = 4 and k = 2
Time taken by A, working alone = 6 4
2
1 2
x
k
1 k
=
+
=
|
|
.
|

\
| +
days
2. A alone would take 7 days more to complete the work than if both A and B worked
together. If B worked alone, he took
4
3
1 days more to complete the job both worked
together. What time would they take if both A and B worked together?
Sol: Here, a = 7, b =
4
7
4
3
1 =
Time taken by A and B working together = ab days =
2
7
4
7
7 = days.
3. A is four times as good a workman as B and takes 18 days less to do a piece of work
than B takes. Find the time in which B alone can complete the work.

Sol: Here, k = 4 and l = 12
Time taken by B working alone =
1 k
kl

days = 16
1 4
12 4
=

days.
4. A can do
4
5
of a work in 15 days. In how many days he can finish
6
1
of the work?

Sol: Here, a = 5, b = 4, c = 1, d = 6, X = 15
Required time = 2
6 5
15 1 4
d a
X c b
=


days.
5. If 12 persons can complete th
5
1
of a work in 4 days, then find the number of persons
required to complete the remaining work in 16 days.

Sol: Here M
1
= 12, D
1
= 4, W
1
=
5
1
and M
2
=?, D
2
= 16, W
2
=
5
4

We have,
1 2 2 2 1 1
W D M W D M =
12 x 4 x
5
4
= M
2
x 16 x
5
1



M
2
= 10.

6. If 5 persons can cut 15 trees in 6 days working 10 hours a day. Then, in how many
days can 12 persons cut 36 trees working 6 hours a day.

Sol: Here M
1
= 5, D
1
= 6, W
1
= 15, t
1
= 10
M
2
= 12, D
2
=?, W
2
= 36, t
2
= 6
We have,
1 2 2 2 2 1 1 1
W t D M W t D M =
5 x 6 x 10 x 36 = 12 x D
2
x 6 x 15
D
2
= 10 days.


7. 8 men or 12 women can do a work in 21 days. In how many days, 5 men and 3 women
would complete the work?

Sol: Here, a = 8, b = 12, c = 5, d = 3 and n = 21
Required no. of days = 24
84
12 8 21
) 3 ( 8 ) 5 ( 12
12 8 21
ad bc
nab
=

=
+

=
|
|
.
|

\
|
+
days.




PIPES AND CISTERNS

Pipes: Generally the pipes are connected to tank or cistern and are used to fill or empty the
tank.

Inlet: A pipe connected with a tank or a cistern that fills it is known as inlet.

Outlet: A pipe connected with a tank or cistern emptying it is known as outlet.

Note:

1. Pipes and Cistern problems are similar to those on Time and Work.
2. The only difference here is, the work done is in terms of filling or emptying a cistern
and the time taken by a pipe or leak (Crack) to fill or empty a cistern respectively.
3. Generally, the time taken to fill a cistern is taken as positive and the time taken to
empty a cistern is taken as negative.
4. The amount of work done i.e., filling or emptying a cistern is generally taken as unity,
unless otherwise specified.

Formulae:

1. If an inlet can completely fill the empty tank in X hours, the part of the tank filled in 1
hour = .
X
1

2. If an outlet can empty the full tank in Y hours, the part of the tank emptied in 1 hour
= .
Y
1

3. If both inlet and outlet are open, net part of the tank filled in 1 hour = .
Y
1
X
1

4. Two pipes A and B can fill or empty a cistern in X and Y hours respectively, while
working alone. If both the pipes are opened together, then the time taken to fill or
empty the cistern = . hours
Y X
XY
+



5. Three pips A, B and C can fill a cistern in X, Y and Z hours respectively, while working
alone. If all the three pipes are opened together, the time taken to fill the cistern =
. hours
ZX YZ XY
XYZ
|
|
.
|

\
|
+ +

Note: This type of formulae can be generated by replacing negative sign wherever a
pipe starts emptying the cistern instead of the standard positive sign.
6. Two pipes A and B can fill a cistern in X hours and Y hours, respectively. There is also
an outlet C. if all the three pipes are opened together, the tank is full in Z hours. The
time taken by C to empty the full tank = . hours
XY ZX YZ
XYZ
|
|
.
|

\
|
+

7. A tank takes X hours to be filled by a pipe. But due to a leak, it is filled in Y hours. The
amount of time in which the leak can empty the full tank = . hours
X Y
XY
|
|
.
|

\
|


8. A cistern has a leak which can empty it in X hours. A pipe which allows Y litres of
water per hour into the cistern is turned on and now the cistern is emptied in Z hours.
The capacity of the cistern is . litres
X Z
XYZ
|
|
.
|

\
|


9. One fill pipe A is k times faster than the other fill pipe B.

a) If B can fill a cistern in x hours, then the time in which the cistern will be full, if
both the full pipes are opened together, is . hours
1 k
x
|
|
.
|

\
|
+

b) If A can fill a cistern in y hours, then the time in which the cistern will be f ull, if
both the full pipes are opened together, is . yhours
1 k
k

|
|
.
|

\
|
+

10. If one fill pipe A is k times faster and takes x minutes less time than the other fill pipe
B, then

a) A will fill the cistern in
|
|
.
|

\
|
1 k
x
minutes.
b) B will fill the cistern in
|
|
.
|

\
|
1 k
kx
minutes.
c) The time taken to fill a cistern, if both the pipes are opened together is
|
|
.
|

\
|

2
) 1 k (
kx
minutes.

SOLVED EXAMPLES

1. A pipe can fill a tank in 4 hours. Find the part of tank filled in one hour.

Sol: The part of tank filled in 1 hour =
4
1
.
2. A pipe can fill a tank in 20 minutes. Find the time in which
5
1
part of the tank will be
filled.

Sol: The part of the tank filled in 1 minute =
20
1

So,
5
1
part of the tank is filled in
5
1
20 = 4 minutes.



3. A pipe can empty a cistern in 20 minutes. Find the time in which
5
2
part of the cistern
will be emptied.

Sol:
5
2
part of the cistern is emptied in 8
5
2
20 = minutes.

4. A pipe can empty a cistern in 20 hours. Find the part of the cistern emptied in 4 hours.

Sol: The part of the cistern emptied in 1 hour =
20
1

So, the part of the cistern emptied in 4 hours =
5
1
20
1
4 = .

5. A tap can fill a cistern in 8 hours and another can empty it in 12 hours. If both the
taps are opened simultaneously, find the time in hours to fill the cistern.

Sol: Here, X = 8 and Y = 12
Part of the cistern filled in 1 hour = .
24
1
12
1
8
1
Y
1
X
1
= =
Total time taken to fill the cistern = 24 hours.

6. Two pipes A and B can fill a cistern in 20 and 30 minutes, respectively. If both the
pipes are simultaneously then find how long will it take to fill the cistern?


Sol: Here, X = 20 and Y = 30
Part of the cistern filled by (A + B) in 1 minute =

12
1
60
5
30
1
20
1
30
1
20
1
= = + = +
Hence, both the pipes together will fill the cistern in 12 minutes.

7. Two pipes A and B can separately fill a cistern in 4 hours and 6 hours respectively,
while a third pipe C can empty it in 3 hours. In what time will the cistern be full, if all
the pipes are opened together?

Sol: Here, X = 4, Y = 6 and Z = - 3
So, the cistern will be full in 12
6
72
) 3 4 ( ) 3 6 ( ) 6 4 (
3 6 4
= =
|
|
.
|

\
|
+ +

hours.

8. Two taps A and B can fill a cistern in 15 and 30 minutes respectively. There is a third
exhaust tap C at the bottom of tank. If all taps are opened at the same time, the
cistern will be full in 25 minutes. In what time can exhaust tap C empty the cistern
when full?

Sol: Here, X = 15, Y = 30 and Z = 25
C can empty the full tank in
|
|
.
|

\
|
+ XY ZX YZ
XYZ

=
|
|
.
|

\
|
+

) 45 15 ( ) 45 30 ( ) 30 15 (
25 30 15
minutes
10
1125
25 30 15
=

= minutes.

9. A pipe can fill a tank in 6 hours. Due to leakage in the bottom, it is filled in 12 hours.
If the tank is full, how much time will the leak take to empty it?



Sol: Here, X = 6 and Y = 12
So, the time taken by the leak to empty the full tank = hours
X Y
XY
|
|
.
|

\
|


= . hours 12
6 12
12 6
=
|
|
.
|

\
|



10. A leak in the bottom of a tank can empty the full tank in 3 hours. An inlet pipe fills
water at the rate of 2 litres per minute. When the tank is full, the inlet is opened and
due to leak, the tank is empty in 5 hours. Find the capacity of the tank.

Sol: Here, X = 3, Y = 2 x 60 = 120 and Z = 5
So, the capacity of the tank = 720
3 5
4 120 3
litres
X Z
XYZ
=
|
|
.
|

\
|


=
|
|
.
|

\
|

litres.

11. One fill pipe A is 2 times faster the second fill pipe B. If A can fill a cistern in 9
minutes, then find the time when the cistern will be full if both fill pipes are opened
together.

Sol: Here, k = 2 and y = 9
So, Cistern will be full in 6 9
1 2
2
y
1 k
k
=
+
=
|
|
.
|

\
|
+
minutes.

12. One fill pipe A is 7 times faster the second fill pipe B takes 72 minutes less than the fill
pipe B. When will the cistern be full if both fill pipes are opened together?

Sol: Here, k = 7 and x = 72, Cistern will be full in 14
) 1 7 (
72 7
) 1 k (
kx
2 2
=

=
|
|
.
|

\
|

minutes.
TIME AND DISTANCE


Speed: The speed of an object is the distance covered by it in a unit time interval. It is
obtained by dividing the distance covered by the object, by the time it takes to cover the
distance.
Thus, Speed =
taken time
travelled Distance

Note:
1. The terms, time and distance are related to the speed of a moving object.

2. If the time taken is constant, the distance travelled is proportional to the speed i.e. more
the speed; more the distance travelled in the same time.

3. If the speed is constant, the distance travelled is proportional to the time taken i.e. more
the distance travelled; more the time taken at the same speed.

4. If the distance travelled is constant, the speed is inversely proportional to the time taken
i.e. more the speed; less the time taken for the same distance travelled.

Formulae:
1. Speed =
Time
Distance


2. Distance = Speed x Time



3. Time =
Speed
Distance

4. x km/hr =
|
.
|

\
|

18
5
x m/sec
5. x m/sec =
|
.
|

\
|

5
18
x km/hr
6. If A covers a distance d
1
km at a speed s
1
km/hr and then d
2
km at s
2
km/hr, then the
average speed during the whole journey is
Average speed =
1 2 2 1
2 1 2 1
d s d s
) d d ( s s
+
+
km/hr
7. If A goes from X to Y at s
1
km/hr and comes back from Y to X at s
2
km/hr, then the
average speed during the whole journey is
Average speed =
2 1
2 1
s s
s s 2
+
km/hr

8. A goes from X to Y at s
1
km/hr and returns back from Y to X at s
2
km/hr. If he takes T
hours in all, the distance between A and B is km
s s
s s
T
2 1
2 1
|
|
.
|

\
|
+
.
9. A and B start at the same time from two points P and Q towards each other and after
crossing they take T
1
and T
2
hours in reaching Q and P respectively, then speed
1
2
T
T
speed s B'
speed s A'
=
10. If the new speed is
b
a
of the original speed, then the change in time taken to cover the
same distance is given by
Change in time = time original 1
a
b

|
|
.
|

\
|

11. A body covers a distance d in time T
1
with speed s
1
, but when it travels with speed s
2

covers the same distance in time T
2
then


time of Difference
speed of Difference
T
s
T
s
d
speed of Product
1
2
2
1
= = =
Note: By equating any two of the above, we can find the unknowns as per the given question.

12. A train travels a certain distance at a speed of s
1
km/hr
without stopping and it covers the same distance at a speed
of s
2
km/hr with stopping then
The stopping time per hour =
. hr
s
s s
stoppings without speed
speed of Difference
1
2 1
|
|
.
|

\
|
=

13. If a train overtakes a pole or a man or a milestone, then the
distance covered in overtaking = Length of the train.

14. If a train overtakes a bridge or tunnel or a platform or
another train, then the distance covered = Sum of the two
lengths.



15. Relative Speed: If two trains of lengths L
1
km and L
2
km,
respectively are traveling in the same direction at s
1
km/hr
and s
2
km/hr respectively such that s
1
> s
2
, then their
a. Relative speed = s
1
s
2
.
b. Time taken by the faster train to cross the slower train =
|
|
.
|

\
|

+
2 1
2 1
s s
L L
hr.
16. Relative Speed: If two trains of lengths L
1
km and L
2
km,
respectively are traveling in the opposite direction at s
1

km/hr and s
2
km/hr respectively then their

a. Relative speed = s
1
+ s
2
.
b. Time taken by the faster train to cross each other =
|
|
.
|

\
|
+
+
2 1
2 1
s s
L L
hr.
17. Two trains of lengths L
1
m and L
2
m run on parallel tracks.
When running in the same direction, the faster train passes
the slower one in T
1
seconds, but when they are running in
opposite directions with the same speeds as earlier, they
pass each other in T
2
seconds then
a. Speed of the faster train =
|
|
.
|

\
|
+
+
2 1
2 1
T
1
T
1
2
L L
m/s.
b. Speed of the slower train =
|
|
.
|

\
|

+
2 1
2 1
T
1
T
1
2
L L
m/s.
18. A train starts from a place at s
1
km/hr and another fast train
starts from the same place after T hours at s
2
km/hr in the
same direction. Then,

a) The distance from the starting place at which both the
trains will meet is given by
|
|
.
|

\
|


1 2
2 1
s s
T s s
km.
b) The time after which the two trains will meet is given
by
|
|
.
|

\
|

1 2
1
s s
T s
hr.

19. The distance between two stations A and B is d km. A train
starts from A to B at s
1
km/hr. T hours later another train
starts from B to A at s
2
km/hr. Then,
a) The distance from the A at which both the trains will
meet is given by
|
|
.
|

\
|
+
+
2 1
2
1
s s
T s d
s km.


b) The time after which the two trains will meet is given
by
|
|
.
|

\
|
+
+
2 1
2
s s
T s d
hr.
20. Two trains start simultaneously from the stations A and B
towards each other each other with speed s
1
km/hr and s
2

km/hr, respectively. When they meet it is found that the
second train had travelled d km more than the first. Then the
Distance between two trains =
|
|
.
|

\
|

+
1 2
2 1
s s
s s
d km.

SOLVED EXAMPLES

1. Find the speed of a train which covers a distance of 160 km in 4 hours.
Sol: Speed = km/hr. 40
4
160
Time
Distance
= =
2. How long does a train 200 m long running at the rate of 80 km/hr take to cross a
telegraphic pole?

Sol: We know, in crossing the telegraphic pole, the train must travel its own length.
So, Distance travelled = 200 m
Speed = 80 km/hr = m/sec
9
200
m/sec
60 60
1000 80
=


So, time taken to cross the pole = seconds. 9
9
200
200
=
3. A train running at a speed of 72 km/hr passes a pole on the platform in 15 seconds.
Find the length of the train in metres.
Sol: Speed of the train = 72 km/hr
= m/sec 20
18
5
72 =
Length of the train = Speed of the train x time taken
= 20 x 15 = 300 m.

4. A ship sails to Vizag at a speed of 10 knots/hr and sails back to the same point at the
rate of 15 knots/hr. Find the average speed for the whole journey.

Sol: Here s
1
= 10 and s
2
= 15
Average speed =
2 1
2 1
s s
s s 2
+
km/hr
= knots/hr. 12
15 10
15 10 2
=
+



5. Sheela started to a bakery with the speed of 5 km an hour and returns with a speed of
3 km/hr. if she takes 8 hours in all, find the distance in km between the bakery and
her house.

Sol: Here s
1
= 5, s
2
= 3 and T = 8
The distance between the bakery and her house =
|
|
.
|

\
|
+
2 1
2 1
s s
s s
T
= km 15
3 5
3 5
8 =
|
|
.
|

\
|
+

.


6. Sujay starts his journey from Bombay to Kolkata and simultaneously Niteesh starts
from Kolkata to Bombay. After crossing each other they finish their remaining journey
in
4
1
6 hours and 4 hours respectively. What is Niteeshs speed if Sujays speed is 40
km/hr?

Sol:
5
4
25
16
4
25
4
4
1
6
4
T
T
speed s Niteesh'
speed s Sujay'
1
2
= = = = =
So, Niteeshs speed = km/hr. 50 40
4
5
speed Sujays
4
5
= =

7. A bike during its journey travels 20 min at a speed of 15 km/hr, another 25 min at 30
km/hr and another 30 min at 15 km/hr. Find the average speed of the bike.

Sol:T
1
=
60
20
, T
2
=
60
25
, T
3
=
60
30
, s
1
= 15, s
2
= 30 and s
3
= 15
So, Average speed of the bike =
3 2 1
3 3 2 2 1 1
T T T
T s T s T s
+ +
+ +


=
60
30
60
25
60
20
60
30
15
60
25
30
60
20
15
+ +
+ +


=
60
30
60
25
60
20
60
30
15
60
25
30
60
20
15
+ +
+ +


= km/hr. 20
75
1500
75
450 750 300
= =
+ +


8. By walking at
4
3
of her usual speed, Rani is 5 minutes late to the conference. Find her
usual time to cover the distance.

Sol: Here, change in time = 5 and
4
3
b
a
=
We have, change in time = time original 1
a
b

|
|
.
|

\
|


original time =






9. Two scooterists do the same journey by traveling at the rates of 9 km/hr and 8 km/hr
respectively. Find the length of the journey when one takes 20 minutes longer than the
other.

Sol: Here, change in speed = 9 8 = 1
Product of speed = 9 x 8 = 72
minutes. 15
1
3
4
5
1
a
b
time in change
=
|
|
.
|

\
|

=
|
|
.
|

\
|




Difference of time = 20 min =
60
20

Length i.e. distance, d =?

We have,

km. 24
60
20
72 d
60
20
1
d
72
time of Difference
speed of Difference
d
speed of Product
= =
= =


10. Without stopping, a train travels certain distance with an average speed of 100 km/hr
and with stoppages; it covers the same distance with an average speed of 80 km/hr.
How many minutes per hour the strain stops?

Sol: Here, s
1
= 100 and s
2
= 60
Stoppage time/hr = min. 12 hr
5
1
100
80 100
s
s s
1
2 1
= =

=
|
|
.
|

\
|


11. A train 500 m long crosses a pole in 6 seconds. Find the speed of the train in km/hr?

Sol: Speed of the train = m/s
6
500
pole the crossing in taking time
train the of Length
=
km/hr 300
5
18
6
500
= = .

12. A train 150 m long passes a bridge in 24 seconds moving with a speed of 54 km/hr.
Find the length of the bridge.

Sol: Speed of the train =
bridge the crossing in taking time
bridge the of Length train the of Length +

m. 210 150 360 bridge the of Length
15 24 bridge the of Length 150
24
bridge the of Length 150
54
18
5
= =
= +
+
=


13. A train 120 m long is running with a speed of 68 km/hr. In what time will it pass a
man walking at 4 km/hr in the opposite direction to that of train?

Sol: Here, L
1
= 120, L
2
= 0, s
1
= 68, s
2
= 4
L
1
+ L
2
= 120 + 0 = 120 m
s
1
+ s
2
= 68 + 4 = 72 km/hr = m/sec. 20
18
5
72 =
Required time = sec. 6
20
120
s s
L L
2 1
2 1
= =
|
|
.
|

\
|
+
+


14. Two trains of length 120 m and 80 m running on parallel tracks in the same direction
with a speed of 48 km/hr and 50 km/hr respectively. In what time will they pass each
other?

Sol: Here, L
1
= 120, L
2
= 80, s
1
= 48, s
2
= 50
L
1
+ L
2
= 120 + 80 = 200 m
s
1
- s
2
= 50 - 48 = 2 km/hr = m/sec.
18
5
2


Required time = sec. 360
5 2
18 200
18
5
2
200
s s
L L
1 2
2 1
=

=
|
|
.
|

\
|

+


15. Two trains of lengths 213 m and 205 m run on parallel tracks. When running in the
same direction the faster train crosses the slower one in
2
1
9 seconds. When running in
opposite direction with the same speeds, they pass each other completely in
2
1
5
seconds. Find the speed of each train.

Sol: Here, L
1
= 213, L
2
= 205, T
1
=
2
19
and T
2
=
2
11

L
1
+ L
2
= 213 + 205 = 418
T
1
+ T
2
= 15
2
30
2
11
2
19
= = +
T
1
- T
2
= 4
2
8
2
11
2
19
= =
Speed of the faster train =
|
|
.
|

\
|
+
+
2 1
2 1
T
1
T
1
2
L L

= m/s. 60
2
11
2
19
15
2
418
T T
T T
2
418
2 1
2 1
=

=
|
|
.
|

\
| +

Speed of the slower train =
|
|
.
|

\
|

+
2 1
2 1
T
1
T
1
2
L L

= m/s. 16
2
11
2
19
4
2
418
T T
T T
2
418
2 1
2 1
=

=
|
|
.
|

\
|


16. A train starts from Hyderabad at 9 A.M. with a speed of 50 km/hr and another train
starts from there on the same day at 1 P.M. in the same direction with a speed of 70
km/hr. Find at what distance from Hyderabad both the trains will meet and also find
the time of their meeting.

Sol: s
1
= 50, s
2
= 70, T = time from 9 A.M. to 1 P.M. = 4 hours.
Distance of meeting point from Hyderabad =
|
|
.
|

\
|


1 2
2 1
s s
T s s
km
= km 700
50 70
4 70 50
=
|
|
.
|

\
|


.
Time of their meeting = hr 10
20
4 50
hr
s s
T s
1 2
1
=

=
|
|
.
|

\
|

after 1 P.M.
i.e. at 11 P.M. on the same day.

17. Vijay Wada is at a distance of 340 km from Hyderabad. A train starts from Hyderabad
to Vijay Wada at 4 A.M. with a speed of 60 km/hr. Another train starts from Vijay
Wada to Hyderabad at 5 A.M. with a speed of 80 km/hr. At what distance from
Hyderabad will the two trains cross each other and also find the time when they cross
each other?

Sol:s
1
= 60, s
2
= 80, T = time from 4 A.M. to 5 A.M. = 1 hr


Distance of meeting point from Hyderabad = km
s s
T s d
s
2 1
2
1
|
|
.
|

\
|
+
+

= km 180
140
420
60 km
80 60
) 1 80 ( 340
60 = =
|
|
.
|

\
|
+
+
.
Time of their meeting = hr 3
120
420
hr
s s
T s d
2 1
2
= =
|
|
.
|

\
|
+
+
after 4 A.M.
i.e. at 7 A.M.

18. Two trains start at the same time from Chennai and Cochin and proceed towards at a
rate of 85 km/hr and 100 km/hr respectively. When they meet, it is found that one
train has travelled 30 km more than the other. Find the distance between Chennai and
Cochin.

Sol: s
1
= 85, s
2
= 100, d = 30
Distance between Chennai and Cochin = km.
s s
s + s
d
1 2
2 1

= km. 370 185 2
15
185
30
85 100
100 85
30 = = =
|
|
.
|

\
|

+



PROBLEMS ON TRAINS

A train is said to have crossed an object (stationary or moving) only when the last coach
(end) of the train crosses the said object completely.

Hence, the distance covered by the train = length of train + length of object





DIFFERENT TYPES OF OBJECTS
On the basis of various types of objects that a train has to cross, we find the following
different cases:

Case Type of Object Time to cross
+

1.
Object is stationary and is of negligible length,
e.g., train crosses lamp post, pole, standing
man, etc.
t =
train of speed
train of length

2.
Object is stationary and is of some length,
e.g., train crosses bridge, a tunnel, platform, or
another train at rest.
t =
train of speed
) object train ( of length +

3.
Object is moving* and is of negligible length,
e.g., train crosses a running man, a running
car, etc.
t =
) object train ( of speed
train of length


4.
Object is moving* and has some length,
e.g., train crosses another running train.
t =
) object train ( of speed
) object train ( of length

+


(*if the object is moving in opposite direction, then denominator becomes speed of (train +
object).

TWO TRAINS CROSSING EACH OTHER IN BOTH DIRECTIONS

Two trains are crossing each other
Length of one train = L
1

Length of second train = L
2

They are crossing each other in opposite direction in t
1
seconds.
They are crossing each other in same direction in t
2
seconds then,


Speed of Faster train =
(

+
+
2 1
2 1
t
1
t
1
2
L L

Speed of Slower Train =
(

+
2 1
2 1
t
1
t
1
2
L L



SOLVED EXAMPLES

1) A train travelling with uniform speed crosses a platform of length 800 metres in 20 seconds and
a tunnel of length 1.1 kilometer in 25 seconds. What is the length of the train?

Sol: Let the length of the train be x metres.

Speed while crossing the platform =
(x 800)
20
+
m/sec.
Time to cross an object moving in the direction of train =
object of Speed train of Speed
object of Length train of Length

+



Speed while crossing the tunnel =
(x 1100)
25
+

Since the train has uniform speed =
(x 800) (x 1100)
20 25
+ +
=
5 (x + 800) = 4(x + 1100)
x = 4400 400 = 400 metres.
The length of the train is 400 metres.

2) If 460 poles are arranged such that the distance between any two successive poles is 20 metres,
and a train 180 m long crosses them completely in three minutes, what is the speed of the train?

Sol: The total distance covered = distance between 460 poles + length of the train

Speed of the train =
( ) 459 20 180
3 60
+

= 52 m/s

3) The train crosses a man standing on a platform 150 metre long in 10 seconds and crosses the
platform completely in 22 seconds. Find the length of train and speed of train.

Sol: Let L
t
and V
t
be the length and the speed of the train
Using the basic formula t =
t
t
L L
V V
+


For standing man, put L = 0 and V = 0, then, we get
10 =
t
t
L
V
_____________________ (1)
For the stationary platform, put V = 0, and L = 150 (given), then we get
22 =
t
t
L 150
V
+

22 =
t
t t
L 150
V V
+ ___________________ (2)
From (1) & (2),
22 = 10 +
t
150
V

t
t
L
Since 10in1
V
(
=
(


V
t
= 12.5 m/sec
Speed of train = 12.5 metre/sec.

From (1) 10 =
t
L
12.5

L
t
= 12.5
length of train = 12.5 metres.

4) A toy train crosses 210 and 122 metre long tunnels in 25 and 17 seconds respectively. Find the
length of train and speed of train.

Sol: Let L
t
and V
t
be the length of train and speed of train respectively.
Using the basic formula t =
t
t
L L
V V
+


For tunnel, put V = 0

25 =
t
t
L 250
V
+

L
t
+ 210 = 25V
t



And 17 =
t
t
L 122
V
+
________________ (1)

L
t
+ 122 = 17 V
t
__________________ (2)

Subtracting (2) from (1)

210 122 = 8 V
t

V
t
= 11 m/sec
Putting V
t
= 11 in equation (1),
L
t
+ 210 = 25 x 11
L
t
= 275 210 = 65 m

Speed of the train = 11 m/sec.
Length of the train = 65 m

5) A train 75 metres long overtook a man who was walking at the rate of 6 km/hr and crossed him
in 18 seconds. Again, the train overtook a second person in 15 seconds. At what rate was the
second person travelling?

Sol: Let V
t
be the speed of the train and V
2
be the speed of the second person
Using the basic formula t =
t
t
L L
V V
+



For the first man, put L = 0, V = 6 km/hr = 6
5 5
18 3
= m/sec, t = 18 sec.

18 =
t
75
5
V
3

V
t
-
5 75
3 18
=

Vt =
75 5 105
18 3 18
+ =
m/sec.

For the second person, put L = 0, V
t
=
105
18
m/sec.

15 =
2
75
105
V
18


105
18
- V
2
= 5

V
2
=
15 18
18 5

= 3 km/hr.

The speed of the second person is 3 km/hr.


6) Two trains of lengths 190 m and 210 m respectively, are running in opposite directions on
parallel tracks. If their speeds are 40 km/hr and 32 km/hr respectively, in what time will they
cross each other?



Sol: Using basic formula t =
t
t
L L
V V
+


Since L
t
= 190 m, L = 210 m, V
t
= 40, V = -32

V
t
V = 40 (-32) = 72 km/hr

= 72
5
18

= 20 m/sec
Hence t =
190 210
20
+

t = 20 seconds
The two trains cross each other in 20 seconds.

7) When two trains were running in the same direction at 90 km/hr and 60 km/hr respectively, the
faster train crossed a man in the slower train in 27 seconds. Find the length of the faster train.

Sol: It is given that the faster train crosses a man in the slower train in the same direction. It
implies that a train crosses a man moving in the same direction.

Speed of slower train = speed of the man (because the man is in the slower train)
Now, using the basic formula, we get t =
t
t
L L
V V
+

, where L1 = length of faster train



For the man, put L = 0

27 =
t
L
5
(90 60)
18

L
t
= 30
5
18

27 = 225

Hence, the length of the faster train is 225 metres.

BOATS AND STREAMS

Still Water: If the speed of the water in the river is Zero, then the water is called Still water.
(Stationary Water)

Stream: If the water of the river is moving, it is called a stream. (Water in motion)

Upstream: If a boat or a swimmer moves against the stream i.e. in the direction opposite to
that of the stream, it is called upstream.

Downstream: If a boat or a swimmer moves with the stream i.e. along the direction of the
stream, it is called downstream.

Note: If the speed of a boat or a swimmer is given, it usually means the speed in still water.





Formulae:

1. If the speed of a boat or a swimmer be x km/hr and
the speed of the stream or the current be y km/hr,
then
a. Speed of the boat or swimmer downstream = (x + y)
kmph.
b. Speed of the boat or swimmer upstream = (x - y)
kmph.
2.
a. Speed of the boat or swimmer in still water
=
2
1
(Downstream speed + Upstream speed)
b. Speed of the stream
=
2
1
(Downstream speed - Upstream speed)
3. If a man is capable of rowing at the speed of x
km/hr in still water, rows the same distance up and
down a stream which flows at a rate of y km/hr,
then his average speed throughout the journey =

x
) y x )( y x (
water still in rate s Man'
Downstream Upstream +
=

km/hr.
4. A man can row a boat in still water at x km/hr. In a
stream flowing at y km/hr, if it takes t hours more in
upstream than to go downstream for the same
distance, then
The distance =
y 2
t ) y x (
2 2

km.
a. A man rows a certain distance downstream in t
1

hours and returns the same distance upstream in t
2

hours. If the speed of the stream be x km/hr, then
The speed of the man in still water =
|
|
.
|

\
|

+
1 2
1 2
t t
t t
y km/hr.


b. A man rows a boat in still water at x km/hr. In a
stream flowing at y km/hr if it takes him t hours to
row to a place and come back, then
The distance between the two places =
|
|
.
|

\
|

x 2
y x
t
2 2
km.
c. A boat or a swimmer takes n times as long to row
upstream so as to row downstream the river. If the
speed of boat or swimmer be x km/hr and the speed
of stream be y km/hr,
Then .
1 n
1 n
y x
|
|
.
|

\
|

+
=


SOLVED EXAMPLES

1. The speed of a boat in still water is 10 km/hr. If the speed of the stream be 2 km/hr,
then find its downstream and upstream speeds.
Sol: Speed of the boat (x) = 10 km/hr.
Speed of the stream (y) = 2 km/hr.
Downstream speed = x + y = 10 + 2 = 12 km/hr.
Upstream speed = x y = 10 2 = 8 km/hr.

2. A boat is rowed down a river 32 km in 4 hours and up a river 15 km in 3 hours. Find the
speed of the boat and the river.
Sol: Speed of the boat downstream = 8
4
32
= km/hr.
Speed of the boat upstream = 5
3
15
= km/hr.
Speed of the boat =
2
1
(Downstream speed + Upstream speed)
=
2
1
(8 + 5) = 5 . 6
2
13
= km/hr.
Speed of the river =
2
1
(Downstream speed - Upstream speed)
=
2
1
(8 - 5) = 5 . 1
2
3
= km/hr.

3. A man rows a boat at a speed of 16 km/hr in still water to a certain distance upstream
and back to the starting point in a river which flows at 8 km/hr. Find his average speed
for total journey.
Sol: Average speed =
x
) y x )( y x (
water still in rate s Man'
Downstream Upstream +
=


= 12
16
8 24
16
) 8 16 )( 8 16 (
=

=
+
km/hr.
4. A man can row 4 km/hr in still water. If the river is running at 3 km/hr, it takes 12 hours
more in upstream than to go downstream for the same distance. How far is the place?
Sol: The required distance = 10
3 2
12 ) 3 4 (
y 2
t ) y x (
2 2 2 2
=

km.


5. A motorboat covers a certain distance downstream in 8 hours but takes 10 hours to
return upstream to the starting point. If the speed of the stream be 14 km/hr, then find
the speed of the motor boat in still water.

Sol: Speed of the motorboat in still water =
|
|
.
|

\
|

+
1 2
1 2
t t
t t
y km/hr
= 126
8 10
8 10
14 =
|
|
.
|

\
|

+
km/hr.
6. A man can row 8 km/hr in the still water. If the river is running at 4 km/hr, it takes him
10 hours to row to a place and back. How far is the place?
Sol: The required distance =
|
|
.
|

\
|

x 2
y x
t
2 2
km
= 30
16
48 10
8 2
4 8
10
2 2
=

=
|
|
.
|

\
|

km.
7. A man can row at the rate of 16 km/hr in still water. If the time taken to row a certain
distance upstream is 7 times as much as to row the same distance downstream, find the
speed of the current.

Sol: Speed of the man =
|
|
.
|

\
|

+
1 n
1 n
speed of the current
16 =
|
|
.
|

\
|

+
1 7
1 7
speed of the current
So, speed of the current = 12 km/hr.


SIMPLE INTEREST

Read this please:

Raju borrowed Rs.1000 from Ramesh. After some days Raju paid back Rs.1200 to Ramesh.
Means Raju paid Rs.200 excess than the amount he borrowed from Ramesh.
- This excess amount is called interest.
- The total amount of money borrowed by Raju from Ramesh is called the Principal or
Sum.
- The money paid back to Ramesh, which is the combination of both Principal and
interest is called the Amount.
- So, Amount = Principal + Interest

Rate of interest: The interest is usually charged according to a specified term, which is
expressed as some per cent of the principal and is called the rate of interest for the fixed
period of time.
- If the fixed period is a year, the rate of interest is charged annually.
- If the fixed period is six months, the rate of interest is charged semi -annually.
- If the fixed period is three months, the rate of interest is charged quarterly.
- If the fixed period is a month, the rate of interest is charged monthly.

Example: If the rate of interest is 10% per annum, then the interest payable on Rs.100 for
one year is Rs.10.

Simple Interest: When the interest is payable on the principal only, it is called the simple
interest. It is the interest calculated on the principal for the entire period it is borrowed. It is
denoted by S.I.

Example: S.I. on Rs.100 at 10% per annum will be Rs.10 each year.
At the end of one year, the total amount will be Rs.100+10 = Rs.110.
At the end of second year, the total amount will be Rs.100+10+10 = Rs.120 and so on.

Formulae:
1. If P stands for Principal, R the rate per cent per annum, T the number of years, A the
amount and S.I. the simple interest then
S.I. =
100
R T P

2.
T R
. I . S 100
P

=
3. %
T P
. I . S 100
R

=
4.
R P
. I . S 100
= T years
5. Amount,
(

+ =
100
RT
1 P A
6. If a certain sum in T years at R% per annum amounts to Rs.A, then the sum will be
) T R ( 100
A 100
P
+

= .
7. The annual payment that will discharge a debt of Rs.A due in T years at R% per annum =
2
) 1 T ( RT
T 100
A 100

+

.
8. If a certain sum is invested in n types of investments in such a manner that equal
amount is obtained on each investment where interest rates are R
1
,R
2
,.,R
n

respectively and time periods and time periods are T
1
,T
2
,., T
n
respectively, then the
ratio in which the amounts are invested is


n n 2 2 1 1
T R 100
1
.... .......... .......... :
T R 100
1
:
T R 100
1
+ + +
.
9. If a certain sum of money becomes n times itself in T years at S.I., then the rate of
interest per annum is, %
T
) 1 n ( 100
R

= .
10. If a certain sum of money becomes n times itself at R% per annum S.I. in T years, then
100(n 1)
T
R

= years.
11. If a certain sum of money becomes n times itself in T years at a simple interest, then the
time
'
T in which it will become m times itself is given by = T
'

m 1
T
n 1
(

(


years.
12. If the rate of interest (R) changes from R
1
to R
2
and P, T are constant, then
Change in S.I. = ) R R (
100
PT
2 1
.
13. If Principal (P) changes from P
1
to P
2
and R, T are constant, then
Change in S.I. = ) P P (
100
RT
2 1
.
14. If Rate (R) Changes from R
1
to R
2
and time (T) changes from T
1
to T
2
but principal (P) is
constant, then
Change in S.I. = ) T R T R (
100
P
2 2 1 1
.
15. If a debt of Rs.X is paid in n number of installments and if the value of each installment
is a, then the borrowed (debt) amount is given by
2
) 1 n ( n
b 100
Ra
na X

+ = .
Where, R is the rate of interest per annum
b is the no. of installments per year
Note:
b = 1, when each installment is paid yearly.
b = 2, when each installment is paid half-yearly.
b = 4, when each installment is paid quarterly.
b = 12, when each installment is paid monthly.

16. If a certain sum of money P lent out at S.I. amounts to A
1
in T
1
years and to A
2
in T
2

years, then,
1 2
1 2 2 1
T ~ T
T A T A
= P
~
and % 100
T A T A
A ~ A
= R
1 2 2 1
2 1
~


17. If a certain sum of money P lent out for a certain time T amounts to A
1
at R
1
% per annum
and to A
2
at R
2
% per annum, then

2 1
2 1 1 2
R R
R A R A
= P
~
~
and . years 100
R A R A
A A
= T
2 1 1 2
2 1
~
~


18. If an amount P
1
lent at S.I. rate of interest R
1
% p.a. and another amount P
2
at S.I. rate
of R
2
% p.a., then the rate of interest for the whole sum is
(

+
+
=
2 1
2 2 1 1
P P
R P R P
R .

19. If a certain sum of money is lent out in n parts in such a manner that equal sum of
money is obtained as S.I. on each part where interest rates are R
1
,R
2
,,R
n
respectively
and time periods are T
1
,T
2
,..,T
n
respectively, then the ratio in which the sum will be
divided in n parts is given by
.
T R
1
.... .......... .......... :
T R
1
:
T R
1
n n 2 2 1 1


20. If there is a change in principal (P), rate of interest (R) and time (T), then the value of
S.I also changes and is given by


2 2 2
1 1 1
2 2
1 1
2 2 2
1 1 1
2
1
T R P
T R P
P A
P A
T R P
T R P
I . S
I . S


=



=

21. Out of a certain sum P,
x
1
part is invested at R
1
%,
y
1
part is invested at R
2
% and the
remainder
(


y
1
x
1
1 say,
z
1
part at R
3
%. If the annual income from all these investments
is Rs.A, the original sum is given by

(
(
(
(

+ +

=
z
R
y
R
x
R
100 A
P
3 2 1
.


SOLVED EXAMPLES

1. Find the S.I. on Rs.3500 for 3 years at 5% per annum.

Sol: Here, P = Rs.3500, T = 3 years and R = 5%
S.I. = 525. . Rs
100
5 3 3500
=



2. Find the Principal, if S.I. = 624 for 2 years at 3% per annum.

Sol: 0400. 1 . Rs
3 2
624 100
T R
. I . S 100
P =

=

3. At what rate per annum will a sum of Rs.4000 amount to Rs. Rs.6000 in 5 years?

Sol: S.I. = 6000 4000 = 2000

%. 10
5 4000
2000 100
T P
. I . S 100
R =

=

4. In what time will Rs.1500 earn an interest of Rs.150 at 2% per annum?

Sol: . years 5
2 1500
150 100
R P
. I . S 100
T =

=

5. Sunny borrowed Rs.2000 from his friend Praveen at 10% per annum for 5 years. Find the
interest and money returned by Sunny to Praveen.
Sol: S.I. =
2000 5 10
1000
100

=

A = P + S.I. = 2000 + 1000 = Rs.3000

6. What sum will amount to Rs.420 at 2% per annum in
2
1
2 years?
Sol: 400 . Rs
105
420 100
)
2
5
2 ( 100
420 100
)
2
1
2 2 ( 100
420 100
) T R ( 100
A 100
P =

=
+

=
+

=
+

=



7. Find the annual installment that will discharge a debt of Rs.16200 in 5 years at 4% per
annum.
Sol: Annual installment =
2
) 1 T ( RT
T 100
A 100

+


= . 3000 . Rs
540
16200 100
2
80
500
16200 100
2
) 1 5 ( 5 4
) 5 ( 100
16200 100
=

=
+

=

+



8. A sum of Rs.3454 is divided among three such parts that amount obtained on these three
parts of money after 3, 4 and 5 years, respectively at rate of 5% per annum remains
equal. Find such three parts of the sum.

Sol: The three parts will be in the ratio

n n 2 2 1 1
T R 100
1
.... .......... .......... :
T R 100
1
:
T R 100
1
+ + +

=
125
1
:
120
1
:
115
1
) 5 5 ( 100
1
:
) 4 5 ( 100
1
:
) 3 5 ( 100
1
=
+ + +

= 552 : 575 : 600
125
69000 1
:
120
69000 1
:
115
69000 1
=


[Since the L.C.M. of 115, 120 and 125 is 69000.]

Ratio = 552 : 575 : 600
Sum of proportional = 600 + 575 + 552 = 1727
1
st
part = 1200 . Rs 3454
1727
600
=
2
nd
part = 1150 . Rs 3454
1727
575
=
3
rd
part = . 1104 . Rs 3454
1727
552
=

9. A certain sum of money quadruples itself in 6 years S.I. Find the rate percent per
annum.
Sol: %. 50 =
6
) 1 4 ( 100
= %
T
) 1 n ( 100
= R

10. In what time a sum of money will double itself at a rate of S.I. of 6% per annum?

Sol: years
3
2
16
6
4
16 100
6
) 1 2 (
years 100
R
1 n
T = =

=
(

=

11. A sum of money put on S.I. doubles itself in 10 years. In how many years would it
quadruple itself?
Sol: Required Time,
'
T =
4 1
10
2 1
(

(


= 30 years.

12. If simple interest on Rs.500 increases by Rs.20, when the rate % increases by 8% per
annum, find the time.
Sol: Change in S.I. = ) R R (
100
PT
2 1

. years
2
1
T T 40 20 8
100
T 500
20 = = =



13. If the S.I. on Rs.3500 be more than the interest on Rs.2000 by Rs.60 in 4 years, then
find the rate per cent per annum.
Change in S.I. = ) P P (
100
RT
2 1

%. 1 R 1500
100
R 4
60 ) 2000 3500 (
100
R 4
60 = = =

14. If the S.I. on a certain sum at 3% per annum for 5 years is Rs.90 more than the interest
on the same sum for 2 years at 6% per annum. Find the sum.
Sol: Change in S.I. = ) T R T R (
100
P
2 2 1 1

. 3000 . 3
100
90 ) 6 2 5 3 (
100
90 Rs P
P P
= = =

15. A sum of Rs.4 is lent to be paid back in 3 equal monthly installments of Re.1 each. Find
the rate percent.

Sol: Here, X = Rs.4, a = Re.1, n = 3, b = 12, R = ?
2
) 1 n ( n
b 100
Ra
na X formula, Required

+ =


%. 400 R
1600 R 1200
400
R 1200
4
3
1200
R
3 4
2
) 1 3 ( 3
12 100
1 R
) 1 ( 3 4
=
= +
+
=
+ =

+ =


16. Anitha deposits Rs.8000 in S.B.I at 2% per annum and Rs.6000 at 5% per annum in
ICICI bank. Find the rate of interest for the whole sum.
Sol: Required rate,
(

+
+
=
2 1
2 2 1 1
P P
R P R P
R

%.
14
5
1
14000
19000
6000 8000
5 6000 2 8000
= =
+
+
=


17. If a sum of Rs.3800 is divided into two such parts that the S.I. on the first part for
2
1
1
years at the rate of 2% per annum, equals the S.I. on the second part for 4 years at the
rate of 4% per annum, then find two such divisions of the sum.
Sol: Required Ratio =
2 2 1 1
T R
1
:
T R
1

1
st
part: 2
nd
part = 3 : 16
16
1
:
3
1
4 4
1
:
2
1
1 2
1
= =


Sum of proportionals = 16 + 3 = 19
So, 1
st
part = 3200 . Rs 3800
19
16
=
2
nd
part = 600 . Rs 3800
19
3
= .



18. If Rs.50 amounts to Rs.85 in 2 years, what Rs. 340 amount to in 15 years at the same
rate?
Sol: By using,
2 2 2
1 1 1
2 2
1 1
T R P
T R P
P A
P A


=

we get

2
2 2
2
2
85 50 85 R 2
A 340 340 R 15
35 170 35 1
A 340 340 15 A 340 2 15
A 340 30 35 1050
A 1050 340 1390

=

= =

= =
= + =

19. From a certain sum Krishna invested, th
4
1
at 2%, th
5
1
at 4% and the rest at 6%. If the
annual income of Krishna is Rs.1840 from all these investments, then find the original
sum.
Sol: Required formula is,
(
(
(
(

+ +

=
z
R
y
R
x
R
100 A
P
3 2 1

5
1
y
1
,
4
1
x
1
= = and
20
11
20
4 5 20
5
1
4
1
1
y
1
x
1
1
z
1
=

= = =
Original sum =
11
20
6
5
4
4
2
100 1840
+ +



. 000 , 40 . Rs 20
92
100 1840
20
66 16 10
100 1840
20
66
5
4
4
2
100 1840

=

=
+ +

=
+ +

=





COMPOUND INTEREST

Compound Interest: In this method, the interest for each period is added to the principal
before; interest is calculated for the next period. So, the principal grows as the interest is
added to it. It is denoted by C.I.

Formulae:

1. If a principal P is given on C.I. at the rate of interest R% p.a., then the Amount A after t
years is
.
100
R
1 P A
t
(

+ =
2. C.I. = A P
=
(
(

|
|
.
|

\
|
+ =
(

+ 1
100
R
1 P P
100
R
1 P
t t

3. Rate of interest (R) = . a . p % 1
P
A
100
t
1
(
(
(

|
|
.
|

\
|

Note: S.I. and C.I. for 1 year at a given rate of interest per annum are always equal.

4. If the interest is compounded half-yearly, then
a) Amount, .
2 100
R
1 P A
t 2
(

+ =
b) C.I. =
(
(

|
|
.
|

\
|

+ 1
2 100
R
1 P
t 2
.
c) Rate (R) = . a . p % 1
P
A
100 2
2
t
1
(
(
(

|
|
.
|

\
|



5. If the interest is compounded quarterly, then
a) Amount, .
4 100
R
1 P A
t 4
(

+ =
b) C.I. =
(
(

|
|
.
|

\
|

+ 1
4 100
R
1 P
t 4
.
c) Rate (R) = . a . p % 1
P
A
100 4
4
t
1
(
(
(

|
|
.
|

\
|



6. If the interest is compounded n times a year, then
a) Amount, .
n 100
R
1 P A
t n
(

+ =
b) C.I. =
(
(

|
|
.
|

\
|

+

1
n 100
R
1 P
t n
.
c) Rate (R) = . a . p % 1
P
A
100 n
n
t
1
(
(
(

|
|
.
|

\
|





7. If the rate of interest is different for different years, say R
1
%, R
2
%, R
3
% for first, second
and third years respectively, then
Amount, .
100
R
1
100
R
1
100
R
1 P A
3 2 1
|
|
.
|

\
|
+
|
|
.
|

\
|
+
|
|
.
|

\
|
+ =
8. If the time is in the form of fraction, say
z
y
x years, then
Amount, .
100
R
z
y
1
100
R
1 P A
x
|
|
|
|
.
|

\
|
+
|
|
.
|

\
|
+ =
9. The difference between the C.I. and the S.I. on a certain sum of money for 2 years at
R% p.a., is
a) C.I. S.I. =
2
100
R
P
|
|
.
|

\
|
if P and R are given
b) C.I. S.I. =
100 2
. I . S R

if S.I. and R are given


10. The difference between the C.I. and the S.I. on a certain sum of money for 3 years at
R% p.a., is
a) C.I. S.I. =
(
(

|
|
.
|

\
|
+
|
|
.
|

\
|
2 3
100
R
3
100
R
P if P and R are given
b) C.I. S.I. =
(
(

|
|
.
|

\
|
+
|
|
.
|

\
|
100
R
3
100
R
3
. I . S
2
if S.I. and R are given
11. If a certain sum becomes n times in t years at C.I.,
a) Then the same sum becomes n
t
in mt years.
b) Rate of C.I., %. 1 ) n ( 100 R
t
1
(
(

=
12. If a certain sum of money at C.I. amounts to Rs.x in A years and to Rs.y in B years, then
the rate of interest p.a. is
%. 100 1
x
y
R
A B
1

(
(
(

|
|
.
|

\
|
=


13. If a loan of Rs.P at R% C.I. p.a., is to be repaid in n equal yearly installments, then the
value of each installment =
n 2
R 100
100
........
R 100
100
R 100
100
P
. Rs
|
|
.
|

\
|
+
+ +
|
|
.
|

\
|
+
+
|
|
.
|

\
|
+
.





SOLVED EXAMPLES

1. Ravi invested Rs.25000 at C.I. rate 4% p.a., for a period of 3 years. What amount will he
receive at the end of 2 years?

Sol: P = Rs.25000, R = 4%, t = 3 years, A =?



. 60 . 121 , 28 . Rs
25
26
25
26
25
26
25000
100
4
1 25000
100
R
1 P A
3
t
= =
(

+ =
(

+ =

2. Find the C.I. on Rs.2000 for 2 years at 5% p.a.
Sol: P = Rs.2000, R = 5%, t = 2 years, C.I. =?
C.I. =
(
(

|
|
.
|

\
|
+ 1
100
R
1 P
t

=
(
(

|
|
.
|

\
|
=
(
(

|
|
.
|

\
|
+ 1
20
21
2000 1
100
5
1 2000
2 2

. 205 . Rs ) 1025 . 0 ( 2000 ) 1 025 . 1 ( 2000 = = =

3. Ram invested Rs.5548 for 3 years at C.I. and received an amount of Rs.6750 on
maturity. What is rate percent?

Sol: P = Rs.5548, A = Rs.6750, t= 3 years, R =?
Rate of interest (R) = . a . p % 1
P
A
100
t
1
(
(
(

|
|
.
|

\
|

% 1
2774
3375
100 % 1
5548
6750
100
3
1
3
1
(
(
(

|
|
.
|

\
|
=
(
(
(

|
|
.
|

\
|
=

. a . p %.
7
1
7
7
50
14
100
% 1
14
15
100 % 1
14
15
100
3
1
3
= = =
(

=
(
(
(
(

|
|
.
|

\
|
|
|
.
|

\
|
=

4. Find the amount of Rs.16000 in
2
1
1 years at 10% p.a., C.I. payable half-yearly.
Sol: P = Rs.16000, R = 10%, t =
2
1
1 =
2
3
years, A =?
Amount,
t 2
2 100
R
1 P A
(

+ =

. 522 , 18 . Rs 9261 2
8000
) 21 (
16000
20
21
16000
2 100
10
1 16000
3
3
2
3
2
= = =
|
|
.
|

\
|
=
(

+ =


5. Find the C.I. on Rs.8192 at 50% p.a., compounded quarterly for 1 year.

Sol: P = Rs.8192, R = 50%, t = 1 year, C.I. =?
C.I. =
(
(

|
|
.
|

\
|

+ 1
4 100
R
1 P
t 4




4930 . Rs
4096
2465
8192
4096
4096 6561
8192
1
8
9
8192 1
4 100
50
1 8192
4 1 4
= =
|
|
.
|

\
|
=
(
(

|
|
.
|

\
|
=
(
(

|
|
.
|

\
|

+ =


6. Find the C.I. on Rs.2000 at 12% p.a. for 2 months compounded monthly.
Sol: P = Rs.2000, R = 12%, t = 2 months = years
6
1
12
2
= , C.I. =?
C.I. =
(
(

|
|
.
|

\
|

+

1
12 100
R
1 P
t 12


| | 40.2. . Rs 0201 . 0 2000 1 0201 . 1 2000
1
100
101
2000 1
12 100
12
1 2000
2
6
1
12
= = =
(
(

|
|
.
|

\
|
=
(
(
(

|
|
.
|

\
|

+ =



7. Lakshmi invests Rs.50000 in a mutual fund which gives interest at 2% p.a., during first
year, 5% during the second year and 8% during the third year. How much does she get
at the end of the third year?
Sol: P = Rs.50000, R
1
= 2%, R
2
= 5%, R
3
= 8%
Amount at the third year,
|
|
.
|

\
|
+
|
|
.
|

\
|
+
|
|
.
|

\
|
+ =
100
R
1
100
R
1
100
R
1 P A
3 2 1


|
|
.
|

\
|
+
|
|
.
|

\
|
+
|
|
.
|

\
|
+ =
100
8
1
100
5
1
100
2
1 50000
57834. . Rs
25000
28917 50000
25
27
20
21
50
51
50000 =

=
|
|
.
|

\
|
|
|
.
|

\
|
|
|
.
|

\
|
=
8. What will be the C.I. on Rs.93750 for
2
1
2 years at 4% p.a.?
Sol: P = Rs.93750, t=
2
1
2 years, R = 4%.
Required C.I. =
(
(
(
(

|
|
|
|
.
|

\
|
+
|
|
.
|

\
|
+ 1
100
R
z
y
1
100
R
1 P
x


9678 . Rs
31250
3226 93750
1
50
51
25
26
25
26
93750
1
100
4
2
1
1
100
4
1 93750
2
=

=
(

=
(
(
(
(

|
|
|
|
.
|

\
|

+ |
.
|

\
|
+ =

9. Find the difference between C.I. and S.I. on a sum of Rs.6250 put for 2 years at 4% p.a.
Sol: P = Rs.6250, t = 2 years, R = 4%
For 2 years, C.I. S.I. =
2
100
R
P
|
|
.
|

\
|
if P and R are given
10 . Rs
25
1
25
1
6250
100
4
6250
2
= =
|
|
.
|

\
|
=
10. The difference between C.I. and S.I. on a certain sum of money for 3 years, at 4% p.a.
is Rs.76. Find the sum.


Sol: C.I. S.I. = Rs.76, R = 3%
For 3 years, C.I. S.I. =
(
(

|
|
.
|

\
|
+
|
|
.
|

\
|
2 3
100
R
3
100
R
P if P and R are given
76 =
(
(

|
|
.
|

\
|
+
|
|
.
|

\
|
2 3
100
4
3
100
4
P
76 =
(
(

|
|
.
|

\
|
+
|
|
.
|

\
|
2 3
25
1
3
25
1
P
76 =
(
(

|
|
.
|

\
|
+
|
|
.
|

\
|
2 3
25
1
3
25
1
P
76 =
(


+
25 25 25
) 25 3 ( 1
P
76 =
(

15625
76
P
15625 . Rs P = .

11. Certain sum of money placed at C.I. doubles itself in 3 years. In how many years will it
amount to eight times itself?


Sol: Here, n = 2, t = 3 years and m = 3
Then the same sum becomes
m
n in mt years = 3 x 3 = 9 years.

12. At what percent C.I. does a sum of money become eight-fold in 3 years.
Sol: Required Rate percent is, %. 100 1 ) 8 ( 100 1 ) n ( 100 R
3
1
t
1
=
(
(

=
(
(

=
13. A certain sum of money at C.I. amounts to Rs.800 in two years and to Rs.1152 in four
years. Find the rate of interest per annum.
Sol: x = Rs.800, y = Rs.1152, A = 2 and B = 4
Required rate of interest, % 100 1
x
y
R
A B
1

(
(
(

|
|
.
|

\
|
=


% 100 1
400
576
% 100 1
800
1152 2
1
2 4
1

(
(
(

|
|
.
|

\
|
=
(
(
(

|
|
.
|

\
|
=


= %. 20 100
20
20 24
% 100 1
20
24
2
1
2
=
(


=
(
(
(
(

|
|
.
|

\
|

14. If a sum of Rs.18120 is to be paid back in two equal installments at %
3
1
1 per annum,
what is the amount of each installment?
Sol: P = Rs.18120, R = %
3
4
= %
3
1
1
Each installment =
2
R 100
100
R 100
100
P
. Rs
|
|
.
|

\
|
+
+
|
|
.
|

\
|
+



=
2 2
3
304
100
3
304
100
18120
3
4
100
100
3
4
100
100
18120
|
|
|
|
.
|

\
|
+
|
|
|
|
.
|

\
|
=
|
|
|
|
.
|

\
|
+
+
|
|
|
|
.
|

\
|
+

=
(

+
=
|
|
.
|

\
|
+
|
|
.
|

\
|
304
300
1
304
300
18120
304
300
304
300
18120
2


=
604 300
304 304 18120
304
604
304
300
18120

= 9241.60 . Rs
10
304 304
=

.

CLOCKS

Minute Spaces: The circumference of a dial of a clock or watch is divided into 60 equal parts.
These parts are called minute spaces.

Hands: The clock has two hands. 1. Hour hand 2. Minute hand

Hour hand: The hour hand or short hand indicates time in hours.
In an hour, the hour hand covers 5 minute spaces.

Minute hand: The minute hand or long hand indicates time in minutes.
In an hour, the minute hand covers 60 minute spaces.

Note:

In every hour, both the hands coincide once.
In every hour, the hands are straight (points in opposite directions) once. In this position,
the hands are 30 minutes apart.
In every hour, the hands are twice at right angles. In this positi on, the hands are 15 minute
spaces apart.
The minute hand moves through
o
6 in each minute whereas the hour hand moves through
o
2
1
in each minute. Thus, in one minute, the minute hand gains
o
2
1
5 than the hour hand.
When the hands coincide, the angle between them is
o
0 .
When the hands point in opposite directions, the angle between them is
o
180 .
The hands are in the same straight line, when they are coincident or opposite to each other.
So, the angle between the two hands is
o
0 or
o
180 .
The minute hand moves 12 times as fast as the hour hand.

Formulae:

1. The two hands of the clock will be together between H and (H + 1) O clock at
|
|
.
|

\
|
11
H 60
O
clock.
2. The two hands of the clock will be at right angles between H and (H + 1) O clock at (5H
15)
11
12
minutes past H O clock.
3. The two hands of the clock will be in the same straight line but not together between H
and (H + 1) O clock at


(5H 30)
11
12
minutes past H when H > 6,
(5H + 30)
11
12
minutes past H when H < 6
4. Between H and (H + 1) O clock, the two hands of a clock are M minutes apart at (5H M)
11
12
minutes past H O clock.
5. Angle between hands of a clock:
a. When the minute hand is behind the hour hand, the angle between the two hands at
M minutes past H O clock = 30 |
.
|

\
|

5
M
H +
2
M
degrees.
b. When the minute hand is ahead of the hour hand, the angle between the two hands,
at M minutes past H O clock = 30 |
.
|

\
|
H
5
M
-
2
M
degrees.
6. The minute hand of a clock overtakes the hour hand at intervals of M minutes of correct
time. The clock gains or loses in a day by
|
|
.
|

\
|
|
|
.
|

\
|

M
24 60
M
11
720
minutes.


Solved Examples

1. At what time between 6 and 7 O clock are hands of a clock together?

Sol: Here, H = 6
|
.
|

\
|
11
H 60
=
11
8
32
11
360
11
6 60
= =


Hands of a clock are together at
11
8
32 minutes past 6 O clock.
2. At what time between 7 and 8 O clock will the hands of a clock be at right angle?

Sol: Here, H = 7
(5H 15)
11
12
= (5 x 7 15)
11
12

=
11
6
54 and
11
9
21
Hands of a clock are at right angle at
11
6
54 minutes past 7 and
11
9
21 minutes past 7.

3. Find at what time between 3 and 4 O clock will the hands of a clock be in the same
straight line but not together.

Sol: Here, H = 3 < 6
(5H + 30)
11
12
= (5 x 3 + 30)
11
12
=
11
1
49
11
540
= .
The hands will be in the same straight line but not together at
11
1
49 minutes past 3
O clock.

4. Find the time between 5 and 6 O clock when the two hands of a clock are 5 minutes apart.

Sol: Here, H = 5 and M = 5
(5H M)
11
12
= (5 x 5 5)
11
12
=
11
8
32 and
11
9
21


The hands will be 5 minutes apart at
11
8
32 past and
11
9
21 past 5 O clock.

5. Find the angle between two hands of a clock at 25 minutes past 8 O clock.

Sol: Here, H = 8 and M = 25
The required angle = 30 |
.
|

\
|

5
M
H +
2
M
degrees
= 30
|
|
.
|

\
|

5
25
8 +
2
5
degrees
=
o
5 . 102 =
2
205
.
6. The minute hand of a clock overtakes the hour hand at intervals of 45 minutes. How much
a day does the clock gain or lose?

Sol: Here, M = 45
The clock gains or loses in a day by
|
|
.
|

\
|
|
|
.
|

\
|

M
24 60
M
11
720
minutes
=
|
|
.
|

\
|
|
|
.
|

\
|

45
24 60
45
11
720

=
|
|
.
|

\
|
|
|
.
|

\
|

45
24 60
45
11
720


=
11
6
654
11
7200
32
11
225
= = minutes.


CALENDARS

- Here you mainly deal in finding the day of the week on a particular given date.
- The process of finding this depends on the number of odd days.
- Odd days are quite different from the odd numbers.

Odd Days: The days more than the complete number of weeks in a given period are called
odd days.

Ordinary Year: An year that has 365 days is called Ordinary Year.

Leap Year: The year which is exactly divisible by 4 (except century) is called a leap year.

E.g. 1968, 1972, 1984, 1988 and so on are the examples of Leap Years.
1986, 1990, 1994, 1998, and so on are the examples of non leap years.

Note: The Centuries divisible by 400 are leap years.

Important Points:

An ordinary year has 365 days = 52 weeks and 1 odd day.
A leap year has 366 days = 52 weeks and 2 odd days.
Century = 76 Ordinary years + 24 Leap years.
Century contain 5 odd days.
200 years contain 3 odd days.
300 years contain 1 odd day.
400 years contain 0 odd days.
Last day of a century cannot be Tuesday, Thursday or Saturday.
First day of a century must be Monday, Tuesday, Thursday or Saturday.

Explanation:

100 years = 76 ordinary years + 24 leap years
= 76 odd days + 24 x 2 odd days
= 124 odd days = 17 weeks + 5 days
100 years contain 5 odd days.

No. of odd days in first century = 5
Last day of first century is Friday.

No. of odd days in two centuries = 3
Wednesday is the last day.

No. of odd days in three centuries = 1
Monday is the last day.

No. of odd days in four centuries = 0
Sunday is the last day.

Since the order is continually kept in successive cycles, the last day of a century cannot be
Tuesday, Thursday or Saturday.
So, the last day of a century should be Sunday, Monday, Wednesday or Friday.
Therefore, the first day of a century must be Monday, Tuesday, Thursday or Saturday.

Working Rules:

Working rule to find the day of the week on a particular date when reference day is given:



Step 1: Find the net number of odd days for the period between the reference date
and the given date (exclude the reference day but count the given date for counting
the number of net odd days).

Step 2: The day of the week on the particular date is equal to the number of net odd
days ahead of the reference day (if the reference day was before this date) but
behind the reference day (if this date was behind the reference day).

Working rule to find the day of the week on a particular date when no reference day is
given

Step 1: Count the net number of odd days on the given date

Step 2: Write:

For 0 odd days Sunday
For 1 odd day Monday
For 2 odd days Tuesday
. . . .
. . . .
. . . .
For 6 odd days - Saturday


SOLVED EXAMPLES

1. If 11
th
January 1997 was a Sunday then what day of the week was on 10
th
January 2000?

Sol: Total number of days between 11
th
January 1997 and 10
th
January 2000
= (365 11) in 1997 + 365 in 1998 + 365 in 1999 + 10 days in 2000
= (50 weeks + 4 odd days) + (52 weeks + 1 odd day) +
(52 weeks + 1 odd day) + (1 week + 3 odd days)

Total number of odd days = 4 + 1 + 1 + 3 = 9 days = 1 week + 2 days

Hence, 10
th
January, 2000 would be 2 days ahead of Sunday i.e. it was on Tuesday.

2. What day of the week was on 10
th
June 2008?

Sol: 10
th
June 2008 = 2007 years + First 5 months up to May 2008 + 10 days of June

2000 years have 0 odd days.
Remaining 7 years has 1 leap year and 6 ordinary years 2 + 6 = 8 odd days

So, 2007 years have 8 odd days.
No. of odd days from 1
st
January 2008 to 31
st
May 2008 = 3+1+3+2+3 = 12
10 days of June has 3 odd days.
Total number of odd days = 8+12+3 = 23

23 odd days = 3 weeks + 2 odd days.
Hence, 10
th
June, 2008 was Tuesday.

MENSURATION: 2D

Types of Plane Figures

1. Triangle 2. Quadrilateral 3. Polygon 4. Circle 5. Sector of a circle
6. Rectangular Paths 7. Circular paths



I. Triangle

(a). Any triangle
a, b and c are three sides of the triangle; h is the
altitude and AC is the base.

Perimeter (P) : P = a + b + c = 2s

Area (A) : A =
2
1
base altitude =
2
1
any side
length of r dropped on that side =
) c s ( ) b s ( ) a s ( s


(b). Equilateral

a is the length of each side

Perimeter (P) : P = 3a

Area (A) : A =
4
3
a
2


(c). Right-angled

b, c are the lengths of the two legs

Perimeter (P) : P = a + b + c = 2s

Area (A) : A =
2
1
product of two legs


(d). Isosceles

a is the length of two equal sides
b is the base
BD is the perpendicular dropped on base such that it divides the
base equally AD = CD =
2
b


Perimeter (P) : P = 2a+b

Area (A) : A =
2 2
b a 4
4
b


(e). Right-angled Isosceles
Perimeter (P) : P = 2 a ( 2 +1)
Area (A) : A =
2
1
(a)
2



B
A C
a
a
a
B
A C
a
b
c
90
o

B
A C
a
D
a
b/2 b/2
B
A
C
a
b
c
h
Hypotenuse - h
a
a



II. Quadrilateral

(a). Any Quadrilateral

AC is the diagonal = d, DE and BF are two perpendicular
drawn on the diagonal (AC) P
1
, and P
2
are the lengths of
the two perpendiculars

Perimeter (P) : P = sum of the four sides.

Area (A) : A =
2
1
d (p
1
+p
2
) =
2
1
any diagonal
(sum of rs drawn on that diagonal)


(b). Rectangle

l = length
b = breadth
d = diagonal

Perimeter (P) : P = 2(l + b) = 2(l+
2 2
l d ) =
2(b+Error! Objects cannot be created from
editing field codes.) = 2 A 2 d
2
+
Area (A): A = l b = l
2 2
l d = b Error!
Objects cannot be created from editing field codes., when p and A are known and l and b are
unknown.
The two values of x will give l and b.


(c). Square

a = length of side
d = diagonal

Perimeter (P) : P = 4a = 2d 2
Area (A) : A = a
2
=
16
p
=
2
d
2 2







(d). Rhombus
a = each side
d
1
= one diagonal
d
2
= another diagonal
h = height

Perimeter (P) : P = 4a = 2
2
2
2
1
d d +
d
A
B
C
D
E
F
P
1

P
2

d
A B
C D
b
b
l
l
O
D
A
A B
A
C
B
A
d
C
B
A
O
B
A
a
O
B
A
a
O
B
A
a
O
B
A
a
O
B
A
90
o
aO
BA
90
o
aO
BA

A
B
A
C
B
A
D
C
B
A
a
D
C
B
A
a
D
C
B
A
a
D
C
B
A
a
D
C
B
A

d
2

O
a
D
C
B
A
90
o
Oa
DC
BA
90
o
Oa
DC
BA
d
1

h




Area (A) : A =
2
1
d
1
d
2
=
2
1 2 1
2
d
a
2
d
(

=
2
2 2 2
2
d
a
2
d
(

= a h

(e). Trapezium

a and b are two parallel sides, h is the height

Area (A) : A =
2
1
(a + b) h =
2
1
(sum of parallel
sides) (perpendicular distance between parallel sides)

(f). Parallelogram
b is the base
h is the perpendicular distance between the base and its
opposite side

Area (A) : A = b h = base (perpendicular distance
between the base and its opposite sides)
= 2 area of ABD (or BCD)
III. Polygon

Polygon is a n-sided closed figure bounded only by line
segments.

In a polygon if the internal angle at each vertex is less than 180
o
then the polygon is a convex
polygon, else a concave polygon.

Convex Polygon:
i. Area of a regular polygon =
2
1
perimeter
r
distance from the center of the polygon to
any side.
ii. Number of diagonals in a polygon =
2
) 3 n ( n

iii. Sum of all interior angles of a polygon = (2n-4) 90
o

iv. Each interior angle of n-sided regular polygon =
(


n
2 n
180
o

v. Sum of all exterior angles of n-sided regular polygon = 360
o

vi. Each exterior angle of n-sided regular polygon =
n
360



IV. Circle

O is the center of the circle

OA = OC = OB = OD = radius of circle = r
AC = BD = diameter of circle = d = 2r

Circumference (or Perimeter) C = 2 r = d

A B
C
B
D
C
B
a
A
b
a
A
h
b
a
A
A B
C
B
D
C
B
b
a
A
h
b
a
A
A
B
A
C
B
A
D
C
B
A
O


Area of circle A = r
2
=
4
d
2

If C = circumference, A = area then
A =
2
r
C
A
and
4
C
2
=



V. Sector of a circle


Area of sector AOB =
2
o
r
360


Length of the Arc AB = r 2
360

o




VI. Rectangular Paths














Case - I

Pathway is outside the rectangle
The length of rectangle AB = l, Breadth BC = b and , Width of path way = W, then

Area of Pathway = 2W (l+b+2w) (shaded portion)

Case II

Path way is inside the rectangle

Area of Pathway = 2W(l+b-2W) (shaded portion)

VII. Circular Pathway






A
B
O
W
w
M
N
K
K
N
K
M
N
K
L
M
N
K
D
N
K
A
D
N
K
B
A
D
N
K
C
B
A
D
N
K
L
C
B
A
D
N
K
Rectangle
Path
way
Case - I
K
N
K
M
N
K
L
M
N
K
D
N
K
A
D
N
K
B
A
D
N
K
C
B
A
D
N
K
L
C
B
A
D
N
K
W
Case - II
W r
R
W
A
O
C
A
W
r
W
C
A
W
W
O







OAC is a circle of radius = r, there is pathway, outside the circle of width = W
Area of circular pathway = W (2r+W)
When, the pathway is inside the circle,
Area of circular pathway = W (2r - W)


SOLVED EXAMPLES

1. If three sides of a triangle are 5, 6 and 7 cm respectively, find the area of triangle.

Sol: Area of A = s(s a)(s b)(s c)

Now, s =
a b c 5 6 7
2 2
+ + + +
= = 9

Area = 9 (9 5)(9 6)(9 7) 9 4 3 2 =
= 216 6 6 = cm
2
.

2. ABC is an equilateral triangle of side 24 cm. Find the in radius of the triangle.

Sol: In a equilateral triangle, the altitude, median and perpendicular are equal.

AD = 3 /2 x 24 = 12 3

GD (in radius) = 1/3 x 12 3 = 4 3 cm

3. The base and other side of an isosceles triangle is 10 and 13 cm respectively. Find its
area.

Sol: Area of Isosceles A =
2 2
b
4a b
4

Given, base b = 10 Other side a = 13

Area (A) =
2 2
10 10
4 (13) 10 676 100
4 4
=
=
10
4
24 = 60 cm
2
.

4. In a right-angled triangle, the length of two legs are 12 and 5 cm. Find the length of
hypotenuse and its area.

Sol: In a right angled triangle,
(Hypotenuse)
2
= (one leg)
2
+ (other leg)
2

= 12
2
+ 5
2

Hypotenuse =
2 2
12 5 + = 169 = 13 cm.



In a right angled triangle,
Area =
1 2
1 1
(leg) (leg) 12 5
2 2
= = 30 cm
2
.

5. If the perimeter and diagonal of a rectangle and 14 and 15 cm respectively. Find its area.

Sol: In a rectangle,


2
(Perimeter)
4
= (diagonal)2 + 2 x Area ;
2
(14)
4
= (5)
2
+ 2 x Area

2 x Area =
196
4
- 25 Area =
49 25
2

= 12 cm
2
.

6. Find the length of the diagonal and the perimeter of a square plot if its area is 900 square
metres.
Sol: In a square, A =
2 2
d p
2 16
=

(Diagonal)
2
= 2 x Area = 900
Diagonal (d) = 2 900 30 2 = = 42.42 metres

(Perimeter)
2
= 16 x Area = 16 x 900

Perimeter (P) = 16 900 = 120 metres.

7. A field in the shape of a rhombus has the distances between pairs of opposite vertices as
14 m and 48 m. What is the cost (in rupees) of fencing the field at Rs.20 per metre?

Sol: The diagonals are 14 m and 48 m

Sides of rhombus =
2 2
14 48
625
2 2
| | | |
+ =
| |
\ . \ .
= 25

Perimeter of rhombus = 4 x 25 = 100 m.
Cost of fencing the field = 100 x 20 = Rs.2000

8. In a trapezium, the length of parallel sides are 20 and 25 metres respectively and the
perpendicular distance between the parallel sides is 12 metres. Find the area of trapezium.

Sol: One parallel side a = 20 metres. Second parallel side b = 25 metres. Height
(perpendicular distance between a and b) = 12 metres.

Area =
1 1
(a b) h (20 25) 12
2 2
+ = + = 270 m
2
.

9. The distance between a pair of opposite vertices of a quadrilateral is 32 units. The lengths
of the perpendiculars drawn on to this diagonal from the other two vertices are 4 1/3 units
and 6 2/3 units respectively. Find the area (in sq units) of the quadrilateral?

Sol: Area of quadrilateral = 1/2 x 32 x
13 20
3 3
| |
+
|
\ .
= 178 sq units.

A B
D
C


10.






In the above parallelogram ABCD, A Z = x + 30
o
and D Z = x 40
o
, what is the measure
of DCB Z ?

Sol: In a parallelogram, sum of adjacent angles is equal to 180
o


x + 30 + x 40 = 180 x = 95
o

DAB Z = x + 30 = 95 + 30 = 125
o

DCB Z = DAB Z = 125
o

(opposite angles of a parallelogram are equal)


11. In a circle of radius 49 cm, an arc subtends an angle of 36
o
at the centre. Find the length
of the arc and the area of the sector.

Sol: Length of the arc =
2 r 2 22 49 36
360 7 360
H
=

= 30.8 cm

Area of the sector =
2
r 22 49 49 36
360 7 360
H
=

= 754.6 cm
2



12. A rectangular plot of dimensions 13 m x 17 m is surrounded by a garden of width 5 m.
What is the area (in sq m) the garden?

Sol: Let ABCD be the rectangular plot of given dimension. The shaded part is the
surrounding garden. Now, the plot ABCD together with the garden forms another
rectangular form PQRS. Dimensions of PQRS, as can be seen from the diagram, are:

Length PQ = width of garden + AB + width of garden
= 5 + 17 + 5 = 27 m
Similarly, breadth = PS = 5 + 13 + 5 = 23 m
Area of garden = Area of PQRS Area of ABCD
= (27 x 23) (17 x 13) = 621 221 = 440 sq m.

13. There is a rectangular field of length 100 m and breadth 40 m. A carpet of 2 m width is to
be spread from the centre of each side to the opposite side. What is the area of the
carpet?

Sol: Area of the carpet ABCD = 40 m x 2 m = 80 m
2

Area of the carpet EFGH = 100 m x 2 m = 200 m
2

But the common area of two carpets = 2 x 2 = 4m
2

So, area of the carpet = 200 + 80 4 = 276 m
2


14. There is an equilateral triangle of which each side is 3 m. With all the three vertices as
centres, circles with radius 1.5 cm are described (i) Calculate the area common to all the
circles and the triangle. (ii) Find the area of the remaining portion of the triangle.

Sol: (i) Area of each sector =
2
1
r
6
H


So area common to the all the circles and triangle = 3
2 2
1 1
r r
6 2
H = H
=
1 22
1.5 1.5
2 7
= 3.53 m
2


(ii) Area of the shaded portion = Area of the triangle Area common to the triangle and
the circles
But area of the triangle =
2 2
3 3 9 3
a (3)
4 4 4
= = m
2


So area of the shaded portion =
9 3
4
m
2
3.53 m
2
= 3.89 m
2
3.53 m
2
= 0.36 m
2

MENSURATION : 3D

Cuboid :

A right prism with a rectangular base is called a Cuboid.
The sides of the base are length (l) and breadth (b). The height is h.

Lateral Surface Area = 2h(l + b)
Total Surface Area = 2h(l + b) + 2lb = 2(lb + bh + hl)
Longest diagonal =
2 2 2
h b l + +
Volume = lbh


Cube:

If the length, breadth and height of a cuboid are all equal, it is called a cube.
Then, if edge of the cube = a
Longest diagonal = 3 a
Lateral Surface Area = 6a
2

Total surface Area = 6a
2

Volume = a
3


Cylinder :

A cylinder can be considered to be a right prism except that instead of identical
polygons a cylinder has identical circles for its top and base and it has a single
lateral surface also called curved surface, instead of several rectangular surfaces.
The basic measurements are the radius of the base (or top) r and the height h.

Curved Surface (Lateral Surface Area) = 2 rh
Total surface Area = 2 rh+2 r
2
= 2 r(h + r)
Volume = r
2
h

Hollow Cylinder:

The cross section of a hollow cylinder is a ring.
Volume of the material of a hollow cylinder =
h(R
2
-r
2
)
Here R is outer radius and r is inner radius of the
hollow cylinder.

h
l
b
a
a
a
r
h
r
R
h




Cone:
A cone can be formed from the sector of a circle by rolling it and joining
together its two straight edges. If r is the radius of the cone, and R is the
radius of the sector of angle , then
1. r = R
360


2. Relation between r, l and h. (the radius, the slant height and
height) is l
2
= h
2
+r
2

3. Curved Surface area of Cone = prl
4. Total Surface Area = rl + r
2
= r(l + r)
5. Volume =
3
1
r
2
h

Sphere:

All points on the surface of a sphere are at the same distance from
the center of the sphere. This distance is called the radius, r.

Surface Area of Sphere = 4 r
2

Volume of a Sphere =
3
4
r
3

The sphere has only one surface and hence only one surface area.

Hemisphere:

The radius is r.

Curved Surface Area = 2 r
2

Total Surface Area = 2 r
2
+ r
2
= 3 r
2

Volume =
3
2
r
3
4
2
1
3
= r
3




SOLVED EXAMPLES


1. A cuboid is 20 m x 10 m x 8 m. Find the length of diagonal, surface area and volume.

Sol: In a cuboid ,

Diagonal d =
2 2 2
l b h + + =
2 2 2
20 10 8 + + = 23.75
Surface are S = 2 (20 x 10 + 10 x 8 + 8 x 20) = 880 m
2

Volume = l x b x h = 20 x 10 x 8 = 1600 m
3
.

2. A cube has edge 12 m. Find its length of diagonal, surface area and volume.

Sol: In a cube
Diagonal d = Edge x 3 = 12 x 3 = 20.78 m
Surface area S = 6 x (Edge)
2
= 6 x (12)
2
= 864 m
2

Volume V = Edge
3
= (12)
3
= 1728 m
3
.

h
r
r
r


3. The base of a right prism is a regular pentagon of side 18 cm. If the height of the prism is 2/3
rd

of the side of the base, how much is the lateral surface area (in sq cm) of the prism?

Sol: Perimeter of the base of the prism
= number of sides x length of each side
= 5 x 18 = 90 cm.
Lateral surface area of a right prism = (Base perimeter) x (height)
= (90)
2
18
3
| |

|
\ .
= 1080 sq cm

4. If the radius of a sphere is increased by 50%, find the increase percent in volume and the
increase percent in the surface area.

Sol: Let original radius = R. Then new radius =
150 3R
R
100 2
= .
Original volume =
3
4
R
3
H , New volume
3
3
4 3R
9 R
3 2
| |
H = H
|
\ .
.
Increase % in volume =
3
3
19 3
R 100
6
4 R
| |
H
|
H \ .
% = 237.5%

Original surface area = 4 H R
2
. New surface area =
2
2
3R
4 9 R
2
| |
H = H
|
\ .
.
Increase % in surface area =
2
2
5 R
100
4 R
| |
H
|
|
H
\ .
% = 125%.

5. A cylinder with base radius of 8 cm and height of 2 cm is melted to form a cone of height 6 cm.
Find the radius of the cone?

Sol: Let the radius of the cone be r cm.

Then,
2 2
1 8 8 2 3
r 6 8 8 2 r
3 6
| |
H =H =
|
\ .
= 64 r = 8 cm.

6. A brick measures 10 cm x 5 cm x 3 cm. How many bricks will be required for a wall of 100
metre long 6 metre high and 1.5 metre thick?

Sol: Volume of the wall = 100 m x 6 m x 1.5 m = 900 m
3


Volume of one brick =
3
1 1 3 3
m m m m
10 20 100 20000
=
No. of bricks required =
3
900m
3
= 60,00,000

3
20000
m
3


7. What is the maximum length of a pencil which can be inscribed in a box of length 24 units,
breadth 3 units and height 4 units?

Sol: Maximum length in a cuboid is its diagonal

Length of main diagonal is
2 2 2
length breadth height + +


= ( ) ( ) ( )
2 2 2
24 3 2 576 9 4 589 + + = + + = units

8. The height and base-radius of a right circular cone are 10 cm and 24 cm respectively. What is
the area of the curved surface area (in sq cm) if the cone?

Sol: Curved surface area of a cone = H rl,

R and l being radius and slant height.
It is given that height h = 10 cm and radius = 24 cm.

L2 = h2 + r2 = 102 +242
l = 26 (10 and 24 are in the ratio of 5 : 12; hence l will be the 2 x 13 = 26
Hence, curved surface area = H rl = H x 24 x 26 624H sq cm.

También podría gustarte